SlideShare a Scribd company logo
Neonatal presentations
to Emergency
Department
Dr Julia Jaffar, Medical Officer
Supervised by Dr Varsha Atul Shah
Neonatal presentations to
Emergency Department
Summary of presentation:
1. Background
2. Factors associated with Ed attendance among
children included
3. Commonest neonatal presentations to ED
4. Singapore data
5. THE MISFITS-Acronym
6. Conclusions
7. CME-MCQ quiz
Background:
 ED attendances by neonates (≤28 days old) are rising
disproportionately quicker than older infants
 With shorter postpartum hospital stays, most of babies are
discharges after 24 hours in normal vaginal delivery of 72 hours in
LSCS
 Many medical concerns, which traditionally have been managed in
newborn nurseries, are presenting to the emergency department
(ED)
 Emergency Departments (ED) are under increasing pressure, with
rising attendances and admission rates
 Neonates present a challenge due to their non-specific
presentations
 Parents may preferentially bring their children to the ED, even for
non-urgent problems
 The reasons are parental anxiety, perceived advantages of Eds
(resources and expertise), and convenience
You are on call MO in
SGH Emergency on
busy day on Sunday
Newborn well baby born to
LSCS mother,
was discharged on D3 of life
on Saturday, with TCB of 192,
was given routine NNJ advice
to visit ED for repeat TCB next
day.
Presented to ED on busy
Sunday afternoon for routine
TCB check
Questions are:
1. Mother with LSCS,
was waiting in
crowded ED next
day, for routine
check of TCB in
crowded ED?
2. Is its appropriate
use of resources?
3. Can they be better
managed at 24 hours
GP clinic?
4. Any other options?
Case 2: Another
extreme:
2 months old
Exprem, who was
discharged 3 days
ago, was found
lifeless with vomited
blood in bed,
sleeping with
mother, was brought
to ED by ambulance
Questions is
1. Will ED will have enough time to attend
to actual real emergency cases?
2. Is adult ED equipped
and trained with managing such neonates?
2.Is it better to bring them to nearest
Children’s Emergency dept?
Daily attendance figures include all patients
who are presented at the EMD regardless of
urgency of medical conditions
Singaporean
context
Unfortunately the most relevant SG
paper on Neonatal presentations to ED is
not specific to Neonates (classified as <
3 years)
+ the paper focusses on reattendances,
not attendances
“The patient’s age was found to be
inversely proportional to the
reattendance rate at our ED, consistent
with preceding studies”
Unplanned reattendances at the paediatric
emergency department within 72 hours: a
one-year experience in KKH
- Goh G.L et al, SMJ 2016
CGH ED study
-Odds of emergency
attendance in children and
is inversely proportional to
the age of the attendees
Odds were 13 times more
for <10 years than even >
70 years old
Some factors that were reported to be
associated with Ed attendance among
children included
Societal/Community:
1. The lack of a continuous relationship with PCPs
2. Caregivers overestimating the severity of their
children’s illness
3. Caregivers having had prior experience of being
taken to the ED as children
4. Being a single parent with no other adults living
in the household
5. Availability of after-hour care offered by the ED
6. Mistrust of the PCP’s ability to manage
paediatric conditions
Factors associated with Ed attendance
among children included
Parental
Parental anxiety
Perception of convenience, perceived resources
and expertise
Believing the problem was more appropriate for
or might require tests in the emergency
department
Feeling that the care provider would refer the
baby to emergency department anyway
Some factors that were reported to be
associated with Ed attendance among
children included
Neonates
 Neonates exhibit non-specific symptoms with serious
conditions
 Parental anxiety, lack of confidence, education,
experience
 Following discharge advice from nurses/doctors
 Most are born at term, never separated from their mother
and discharged within 48 hours of birth
 Assessment of neonates is challenging
 A vulnerable and high-risk cohort
1st
problem:
UK: ED attendances by neonates
are rising disproportionately
quicker than older infants
In Europe and North America up to
one-third of neonates attending
EDs have no medical issue, and a
high proportion have low acuity
problems requiring no medical
investigation or treatment
Why act?
 EDs are suboptimal environments for assessing
well neonates
 Potential over-investigation & treatment
 Potentially modifiable group of patients whose
problems can be managed away from the ED
 Reducing ED attendances for neonates with benign
conditions may benefit patients, families, and
healthcare systems
Problem Community/Society
Hospital/Doctors/Nurse
s
Babies/Patient
s
Parents
Commonest neonatal
presentations
Nonemergency
 Neonatal Jaundice
 Feeding problems,
vomiting/regurgitation
 Difficulty breathing,
 Noisy breathing
 Excessive crying/Colic
 Irritability
 ?Fever
 Emergency
• Respiratory
• Cardiac
• Shock
• CNS
Most common Diagnoses
 More than half “No significant medical problem”
 Bronchiolitis
 ?Sepsis
Common Neonatal Emergencies
Acronym- The Misfits
THE MISFITS
 T- Trauma (accidental nonaccidental)
 H- Heart Disease/Hypovolemia/Hypoxia
 E- Endocrine (congenital adrenal hyperplasia,
 thyrotoxicosis)
 M- Metabolic (electrolyte imbalance)
 I- Inborn Errors of Metabolism metabolic
emergencies
 S- Sepsis (meningitis, pneumonia, UTI)
 F- Formula mishaps (under or over dilution)
 I- Intestinal catastrophes (volvulus,
 intussusception, NEC
 T- Toxins/poisons
 S- Seizures
Trauma (accidental non-accidental)
 May be a difficult process
 Non-accidental? subtle historical findings and no
 physical exam findings
 Presenting symptoms may be nonspecific
 Early diagnosis of an occult head injury may
 prevent significant long-term morbidity
 An ALTE is often an unrecognized presenting
 symptom of abusive head injuries
Trauma (accidental non-
accidental)
 Infants with ALTE w/o an immediate obvious cause
should be evaluated for head trauma with
neuroimaging
 CT scan, HUS or MRI
 Skull x-rays may not be helpful- significant head
injury w/o skull fracture
 Consider neuroimaging in any non-accidental
injury for other skeletal injuries regardless of
physical examination of the head

Trauma(accidental non-
accidental)
 37 of abused children lt 2 y/o had an occult traumatic injury
 In addition, the ophthalmologic evaluation did not demonstrate retinal
hemorrhages in most of the patients
Trauma(accidental non-accidental)
Management
 Evaluation and stabilization of the ABCs
 Bedside glucose evaluation
 Appropriate temperature regulation
If bruising or known intracranial bleed
 CBC
 Platelet count
 PT/PTT
 Neuroimaging after stabilization
Trauma(accidental non-accidental)
 Admit the patient
 Report injury to appropriate state department for
abuse
 Skeletal survey
 Ophthalmologic exam
Heart Disease and Hypoxia
Cyanotic Heart Disease
Cyanosis requires immediate attention and evaluation
Differential diagnosis
 Respiratory causes
 Infectious causes
 CNS abnormalities
 Toxins
 Cyanotic heart disease
Heart Disease and Hypoxia
Cyanotic Heart Disease
Terrible Ts
 Transposition of the great arteries (TGA)
 Tetralogy of Fallot (TOF)
 Tricuspid atresia (TA)
 Total anomalous pulmonary venous return (TAPVR)
 Truncus arteriosus (TA)
Heart Disease and Hypoxia
Cyanotic Heart Disease
• May not be detected at birth
• Adequately oxygenated blood ? PDA ? systemic
circulation
• PDA functionally closes in the first 10-14 hrs of
life
• Several factors can delay its closure
• Prematurity
• Respiratory distress
• Acidosis
• Hypoxia

Heart Disease and Hypoxia
Cyanotic Heart Disease
 PDA is anatomically closed by 2 weeks of age,
contributing to a delayed detection of cyanotic
heart disease
 100 FiO2
 Non-cardiac disease
 At least 10 increase in O2 saturation
 Cyanotic heart disease
 Minimal change in O2 saturation
Heart Disease and Hypoxia
Cyanotic Heart Disease
• PDA is anatomically closed by 2 weeks of age,
Hyperoxia test
• Initial ABG on R/A
• Repeat ABG after 10-20 minutes of 100 O2
• Cyanotic heart disease? PaO2 will not increase
significantly
• If PaO2 rises above 150 mm Hg, cardiac disease
can generally be excluded
• Failure of PaO2 to rise above 150 mm Hg suggests
a cyanotic cardiac malformation

Heart Disease and Hypoxia
Cyanotic Heart Disease
 During stabilization, the physical exam should
include 4 limb BP and careful
cardiac exam
 A murmur may be audible
 Absence of a murmur does not exclude a cardiac
defect
 CXR EKG should be included in the evaluation
 ECHO is diagnostic
Heart Disease and Hypoxia
Cyanotic Heart Disease
Management
 PGE1
 Bolus of 0.05 mcg/Kg IV
 Drip of 0.05-0.1 mcg/Kg/min
 Secure airway
 Profound apnea is a non-dose dependent
 complication of PGE1
Hypoplastic Left Heart Syndrome
 25 of cardiac deaths during first week of life
 Occurs in both cyanotic and acyanotic forms
 In 15 of cases the FO is intact preventing
 mixing at the atrial level
 Infants with mixing at the atrial level are
 acyanotic
Hypoplastic Left Heart Syndrome
 Pulmonary Edema
 Pallor
 Tachypnea
 Poor perfusion
 Poor to absent peripheral pulses
 Loud single S2
 Gallop rhythm w/o murmur
 Hepatomegaly
 Metabolic acidosis
Hypoplastic Left Heart Syndrome
 EKG
 Small or absent (L) ventricular forces
 CXR
 Moderate cardiomegaly
 Large PA shadow
 ECHO
 Small or slit-like (L) ventricle
 Hypoplastic ascending aorta
Hypoplastic Left Heart Syndrome-
Treatment
 PGE1- systemic blood flow is ductal dependent
 Surgical correction
 Surgical correction
 1st stage
 Norwood procedure
 2nd stage
 Fontan procedure
 Neonatal cardiac transplantation
 Compassionate care may be appropriate in some
instances

Acyanotic Heart Disease-
Congestive Heart Failure
Typically presents with symptoms of CHF
 Tachypnea
 Tachycardia
 Hepatomegaly
 History of poor or slow feeding
 Sweating or color change with feeding
 Poor weight gain
 More gradual clinical decompensation
 May not present until after the first 2-3 weeks
of age
Acyanotic Heart Disease
Congestive Heart Failure
Causes of CHF in Neonates
1. Acyanotic heart disease (VSD, ASD, PDA, CoA)
2. Severe anemia
3. Trauma
4. Sepsis
5. SVT
6. Metabolic abnormalities
7. SLE
8. Thyrotoxicosis
Acyanotic Heart Disease
Congestive Heart Failure
Initial management
Stabilization of the ABCs
CXR
EKG
Labs
FBC
BMP
ABG
ECHO- diagnostic of heart defect
Furosemide-1 mg/Kg IV
Acyanotic Heart Disease
Congestive Heart Failure
Pressors
Dopamine
5-15 mcg/Kg/min IV
Dobutamine
2.5-15 mcg/Kg/min IV
Careful with fluid overloading
Peds. Cardiology consult
Acyanotic Heart Disease-
Supraventricular Tachycardia
Acyanotic Heart Disease
 SVT is the most common neonatal dysrhythmia
 (1/25,000 births)
Signs/symptoms
 Tachycardia
 Poor feeding
 Irritability
 Heart Failure
 Shock
 Heart rate sustained at gt220 bpm with a QRS lt 0.08
seconds
Acyanotic Heart Disease-
Supraventricular Tachycardia
Management
 Stable patient
 Vagal manoeuvres
 Ice to face avoiding the nares
 If unsuccessful
 Adenosine
 50 mcg/Kg rapid IVP (1-2 secs.), increase dose in
50mcg/Kg increments Q2 mins. until return of
sinus rhythm, maximum dose 250 mcg/Kg
Acyanotic Heart Disease-
Supraventricular Tachycardia
Management
 Unstable patient w/o IV access
 Synchronized cardioversion- 0.5-1 J/Kg
 Initial cardioversion should be attempted
pharmacologically if IV access is established and
adenosine is readily available
If unresponsive to adenosine cardioversion
 Amiodorone- 5mg/Kg IV over 30-60 mins.
Acyanotic Heart Disease-
Supraventricular Tachycardia
Procainamide- alternative to amiodorone- 15 mg/Kg IV over 30-60
mins.
 The administration of procainamide and amiodorone
together can lead to hypotension and widening of
the QRS complex
Lidocaine- 1mg/Kg IV
Final option for a wide QRS and should only be
used in consultation with a pediatric cardiologist

Acyanotic Heart Disease-
Supraventricular Tachycardia
 12-lead EKG prior to and after conversion from
SVT to NSR
 Useful diagnostic tool for the cardiologists to
help determine further management
 Consult pediatric cardiologist for further
evaluation
Heart Disease and Hypoxia
Bronchiolitis- Viral
 lower-airway disease caused by RSV 80 of
the time
 Other etiologies include adenovirus,
influenza,
or parainfluenza
 RSV is responsible for 50-90 of bronchiolitis
hospital admissions
 More common in winter and spring seasons,
may
present at any time
Heart Disease and Hypoxia
Bronchiolitis- Viral
Signs/Symptoms
 Rhinorrhea
 Cough
 Congestion
 Wheezing
 Significant respiratory distress
 Apnea may be the only initial symptom
Heart Disease and Hypoxia
Bronchiolitis- Viral
Management
 Infants with severe, prolonged apnea with
bradycardia unresponsive to O2 therapy may need
intubation
 Nebulized racemic epinephrine
or
 Beta-agonist
 The adjunct use of corticosteroids has not been
shown to improve symptoms
 A fever or sepsis evaluation may be part of the
management
Heart Disease and Hypoxia
Bronchiolitis- Viral
Management
 Controversy over the incidence of severe
bacterial infections in infants who have RSV
 The presence of a viral infection doesnt exclude
the possibility of a concomitant UTI
 Consider hospitalization for all RSV() neonates,
especially preemies or all neonates with other
comorbidities
Heart Disease and Hypoxia
Apnea/ALTE
 Apnea
 cessation of respiration for 20 secs. or more, associated
with color change (cyanosis or pallor) or bradycardia
 ALTE
poorly defined term used to describe any event
that is frightening to the observer and is
 characterized by some combination of apnea, color
change, marked change in muscle tone, choking or
gagging
Heart Disease and Hypoxia
Apnea/ALTE
Management depends on history provided by
observers and PE
 Hospitalization for observation and monitoring
 Common differential diagnosis
 Sepsis
 Pneumonia
 RSV
 Hypothermia
 Anemia
Heart Disease and Hypoxia
Apnea/ALTE
 Botulism
 Dysrhythmias
 Acid/base disturbances
 Intracranial haemorrhage
 Meningitis/encephalitis
 Pertussis
 Hypoglycaemia
 Seizures
 GER
 Child abuse
 Inborn errors of metabolism
 Electrolyte abnormalities
Endocrine Emergencies
Congenital Adrenal Hyperplasia
 Most patients diagnosed by newborn screening
 Occasionally diagnosis is missed because of
inadequate blood sample, laboratory error, or
inability to contact the family
Endocrine Emergencies
Congenital Adrenal Hyperplasia
 Autosomal recessive
 Most common is 21-hydroxylase deficiency- 95 of
affected patients
 Inadequate cortisol levels
 Excessive ACTH stimulation
 Adrenal hyperplasia
 Excessive production of adrenal androgens and
testosterone
 ? virilization

Endocrine Emergencies
Congenital Adrenal Hyperplasia
Two forms
• Virilizing form
• Relative aldosterone deficiency
• Mild salt loss
• Adrenal insufficiency tends not to occur unless
under stressful situations
• Salt-losing form
• Absolute aldosterone deficiency
• Adrenal insufficiency under basal conditions
• Manifests in the neonatal period or soon after as
an adrenal crisis

Endocrine Emergencies
Congenital Adrenal Hyperplasia
 11- hydroxylase deficiency
 Less common- 5-8 of cases
 Salt retention
 Volume expansion
 Hypertension

Endocrine Emergencies
Congenital Adrenal Hyperplasia
Management
Labs
 Blood glucose
 Hypoglycaemia
 Serum electrolytes
 Hyponatremia
 Hyperkalaemia
 Hypotension unresponsive to fluids or inotropes
heightens suspicion of CAH

Endocrine Emergencies
Congenital Adrenal Hyperplasia
Management
 Hydrocortisone
 25-50mg/m2 IV
 Treat hypoglycemia
 Hyperkalemia usually responds to fluid therapy
 If patient is symptomatic or with EKG changes
 Calcium chloride
 NaHCO3
 Insulin and glucose
 Polystyrene sulfonate (Kayexalate)
Endocrine Emergencies
Congenital Adrenal Hyperplasia
Management
 Pediatric critical care management
 Endocrinology consultation
Endocrine Emergencies
Thyrotoxicosis
 Hypermetabolic state resulting from excessive
thyroid hormone activity in the newborn
 Usually results from transplacental passage of
thyroid-stimulating immunoglobulin from a mother
with Graves disease
 Rare disorder
 Occurs in 1/70 thyrotoxic pregnancies
 Incidence of maternal thyrotoxicosis in pregnancy
is 1-2/1000 pregnancies

Endocrine Emergencies
Thyrotoxicosis
Clinical presentation
 Fetal tachycardia in the 3rd trimester may be the
first manifestation
 Signs usually apparent within hours from birth
 If mother is on antithyroid medications
presentation may be delayed 2-10 days
 Thyrotoxic signs
 Irritability
 Tachycardia
 Flushing
 Tremor
 Poor weight gain
 Trombocytopenia
 Arrhythmias
Endocrine Emergencies
Thyrotoxicosis
 Initial diagnosis difficult w/o clear history of
Graves disease from mother
 Goiter usually present ? tracheal compression
Labs
 Increased T4, FT4 T3
 Suppressed levels of TSH
Treatment
 Mild
 Close observation

Endocrine Emergencies
Thyrotoxicosis
Moderate
 Lugols solution (iodine)- 1 drop PO Q8H
 Propylthiouracil- 5-10mg/Kg/day in 3 divided doses
 Methimazole- 0.5-1mg/Kg/day in 3 divided doses
Severe
In addition to above meds
 Prednisone- 2mg/Kg/day
 Propranolol for tachycardia- 1-2mg/Kg/day in 2-4 divided doses
 Digitalis may be used to prevent cardiovascular collapse
Inborn Errors of Metabolism
Urea cycle defects
 Ornithine-transcarbamylase deficiency
 Carbamyl phosphate synthetase deficiency
 Transient hyperammonemia of the neonate (unclear
cause)
 Argininosuccinate synthetase deficiency
(citrulinemia)
 Argininosuccinate lyase deficiency
 Arginase deficiency
 N-acetylglutamate synthetase deficiency

Inborn Errors of Metabolism
Amino acid metabolism defects
 MSUD
 Nonketotic hyperglycinaemia
 Hereditary tyrosinemia
 Pyroglutamic academia (5-oxoprolinuria)
 Hyperornithinemia-hyperammonemia-homocitrulinemia
syndrome
 Lysinuric protein intolerance
 Methylene tetrahydrofolate reductase deficiency
 Sulphite oxidase deficiency
Inborn Errors of Metabolism
Organic Acidemias
 Methylmalonic acidemia
 Propionic acidemia
 Isovaleric acidemia
 Multiple carboxylase deficiency
 Glutaric acidemia type II
 HMG-CoA lyase deficiency
 3-Memethylcrotonoyl-CoA carboxylase deficiency
 3-Hydroxyisobutyric acidemia
Inborn Errors of Metabolism
Carbohydrate metabolism defects
 Galactosemia
 Fructose-1,6-biphosphatase deficiency
 Glycogen storage diseases (types IA. IB, II, III
and IV)
 Hereditary fructose intolerance

Inborn Errors of Metabolism
Fatty acid oxidation defects
 Short chain acyl-CoA dehydrogenase deficiency
(SCAD)
 Medium chain acyl-CoA dehydrogenase deficiency
(MCAD)
 Most common (incidence of 1/6,000-10,000)
 Long chain acyl-CoA dehydrogenase deficiency
(LCAD)
 Acyl-CoA deficiency

Inborn Errors of Metabolism-
Metabolic Emergencies
 Often have a delayed diagnosis
 Symptoms may be unrecognized because they are
 uncommon
 Require a high level of suspicion for diagnosis
 Diagnosis should be considered in any infant who
 does not have any other obvious cause for symptoms
Inborn Errors of Metabolism-
Metabolic Emergencies
 Nonspecific symptoms
 Poor feeding
 Vomiting
 FTT
 Tachycardia
 Tachypnoea
 Irritability

Inborn Errors of Metabolism-
Metabolic Emergencies
 More apparent symptoms
 Seizures
 Lethargy
 Hypoglycemia
 Apnea
 Temperature instability
 Acidosis

Inborn Errors of Metabolism-
Metabolic Emergencies
Labs
 Bedside glucose
 FBC
 Basic metabolic panel (BMP)-UECr, Blood sugar, CaPo4
 pH
 Lactate and ammonia levels
 LFTs
 Urine for reducing substances and ketones
 Blood and urine for organic and amino acids
Inborn Errors of Metabolism-
Metabolic Emergencies
Management
 Fluid resuscitation
 IV dextrose to prevent further catabolism
 Admission to hospital
 Genetics consultation

Sepsis
• It is standard of care to complete a full sepsis
workup (FBC, blood culture, urinalysis, urine
culture, CSF culture and analysis, CXR) in a
neonate with a rectal temperature of 38 C

Sepsis
 The use of peripheral WBC count is not
helpful to differentiate febrile neonates with a
more serious bacterial infection from those
w/o
serious bacterial infection
 One study demonstrated that a low peripheral
WBC count increased the odds of bacterial
meningitis
Sepsis
• The urinalysis may be unremarkable in infants
with a culture () UTI
• Approximately 14 of febrile neonates will be
diagnosed with a UTI
• Pediatrics 2000
• McKay Memorial Hospital in Taiwan
• CRP, ESR and U/A imperfect tools in
discriminating for UTI

Sepsis
Broad spectrum antibiotics
 Ampicillin
 50-100mg/Kg IV
 Gentamicin
 2mg/Kg IV
or
 Cefotaxime
 50-100mg/Kg IV
 Acyclovir
 20mg/Kg IV

Sepsis
Neonatal herpes
• Symptoms may be subtle
• No maternal history in 60-80 of women with
unrecognized infection
• Early recognition and treatment with acyclovir
may decrease mortality from 90 ? 31
• Initiate treatment in any infant with
• High fever
• CSF lymphocytosis
• Numerous RBCs in an atraumatic spinal tap
• Seizures
• Known maternal history of HSV infection

Sepsis
 CSF analysis
 Herpes PCR
 Herpes culture
 Elevated LFTs
 Chest x-rays
 Pneumonitis
Formula Mishaps
 Inappropriate mixing of water and powder formula
 Overdilution of concentrated liquid or premixed formula
 Life-threatening electrolyte disturbances or FTT
 Hyponatremia
 Seizures
Intestinal Catastrophes
 Consider pathologic process if vomiting in
newborn period
 Difficult to differentiate between a
life-threatening cause from a mild viral
gastroenteritis or even severe gatroesophageal
reflux
 Initial symptoms may be nonspecific
 Bilious emesis is almost always an ominous sign
 Initiate pediatric surgery consultation

Intestinal Catastrophes-
Malrotation with Midgut Volvulus
• Abnormal rotation of bowel in utero resulting in
an unfixed portion of bowel that may later twist
on itself ? bowel ischemia ? death
• Incidence of 1/5,000 live births
• Usually diagnosed in the first month of life

Intestinal Catastrophes- Malrotation
with Midgut Volvulus
Symptoms
• Bilious emesis
• Poor feeding
• Lethargy
• Shock in more advanced presentations
• Management
• Fluid resuscitation
• NGT placement
• Pediatric surgical consultation

Intestinal Catastrophes- Malrotation
with Midgut Volvulus
• KUBs-Normal
• Signs of small bowel obstruction
• Upper GI series is the gold standard for
diagnosis
• Transverse portion of the duodenum leading to a
fixed ligament of Treitz

Intestinal Catastrophes-
Toxic Megacolon
 Life-threatening presentation of a patient with
Hirschsprung's disease
 Hirschsprung's disease occurs in 1/5,000 live
births
 May be unrecognized because constipation is
common and usually benign
 History of constipation with failure to pass
meconium in the first 24 hours of life is highly
suspicious of Hirschprungs

Intestinal Catastrophes- Toxic
Megacolon
Symptoms
• Poor feeding
• Vomiting
• Irritability
• Abdominal distention
• Hematochezia
• Shock as it progresses to enterocolitis

Intestinal Catastrophes-
Toxic Megacolon
Management
• Stabilization of ABCs
• Fluid resuscitation
• Broad-spectrum antibiotics
• KUB
• Enlarged or dilated section of colon
• Surgical consultation
• Pediatric critical care management in the
presence of enterocolitis

Intestinal Catastrophes-
Necrotizing Enterocolitis
 Classically a disease of premature infants
 May occasionally occur in term neonates after
discharge from WBN
 Symptoms like those of Hirschsprung's
enterocolitis

Intestinal Catastrophes-
Necrotizing Enterocolitis
Management
 Stabilization of ABCs
 Fluid resuscitation
 NGT placement
 Broad-spectrum antibiotics
 Pediatric surgical consultation
 Critical care management

Intestinal Catastrophes-
Hypertrophic Pyloric Stenosis
• Common, incidence of 1/250 live births
• Male/female ratio 4/1
• More common in firstborn male
• Classic metabolic abnormality of hypochloremic,
hypokalemic metabolic alkalosis- now uncommon
• History of nonbilious projectile emesis
immediately after feeding

Intestinal Catastrophes-
Hypertrophic Pyloric Stenosis
 Increased incidence in infants with an early
exposure to oral erythromycin
PE
 Palpable olive structure in the RUQ
 Visible peristaltic waves
 Diagnosis
 USG
 Thickened and lengthened pylorus
 Upper GI
 String sign
Intestinal Catastrophes-
Hypertrophic Pyloric Stenosis
Management
 Surgical is standard
 IV atropine followed by oral atropine shows
satisfactory results
 Stabilization and IV access to replace fluids and
electrolytes
 Osaka, Japan
 Archives of Disease in Childhood 2002
 89 resolution of projectile vomiting with
reduced pyloric muscle thickness
Toxins
 Toxic ingestions are uncommon
 Occasionally the result of a maternal ingestion
in a breastfeeding mother, homeopathic remedies,
or overuse of accepted medications
 Teething gels may be used for the relief of colic
 Benzocaine
 Methemoglobinemia with overuse
Toxins
 Star anise tea
 Relief of infantile colic
 Neurotoxicity
 Unexplained irritability
 Vomiting
 Seizures
 Baking soda
 Used for intestinal gas
 Serious toxicity
 Hospitalization for monitoring and observation

Seizures
 May be difficult to diagnose
 Not acting right
 More somnolent than usual
 Immature cortical development
 May not be tonic-clonic
Commonly
 Lip-smacking
 Abnormal eye or tongue movements
 Pedaling
 Apnea
Seizures
Common causes of neonatal seizures
1st day of life
 Anoxia/hypoxia
 Trauma
 Intracranial haemorrhage
 Drugs
 Infection
 Hypoglycaemia/hyperglycaemia
 Pyridoxine deficiency
Seizures
2nd day of life
 Sepsis
 Trauma
 Inborn errors of metabolism
 Hypoglycaemia
 Hypocalcaemia
 Hyponatremia/hypernatremia
 Hyperphosphatemia
 Drug withdrawal
 Congenital anomalies or developmental brain
disorders
 Benign familial neonatal seizures
Seizures
Day 4 to 6 months of age
 Hypocalcaemia
 Infection
 Hyponatremia/hypernatremia
 Drug withdrawal
 Inborn errors of metabolism
 Hyperphosphatemia
 Congenital anomalies or developmental brain
disorders
 Hypertension
 Benign idiopathic neonatal seizures

Seizures
Management
 Stabilization of ABCs
 Labs
 Bedside glucose level
 Immediate correction of hypoglycemia (lt40mg/dL)
with 2-4mL/Kg D10W may be necessary
 Serum electrolytes
 CBC
 Blood CS
 LFTs

Seizures
Because 5-10 of neonatal seizures are of
infectious etiology, full sepsis work-up
should be performed when patient is
stable
Seizures
Management
• Phenobarbital
• Loading dose 20mg/Kg slow IV push over 10-15
mins, additional 5mg/Kg doses up to 40mg/Kg
• Maintenance of 3-4mg/Kg/day, 12-24 hours after
loading dose

Seizures
Phenytoin
• Loading dose of 15-20mg/Kg IV over 30 minutes
• Maintenance dose of 4-8mg/Kg IV slow push or PO
• Highly unstable in IV solutions
• Avoid using in central lines because of risk of
precipitation
• IM not an option- crystallizes in muscle
Lorazepam - 0.05-0.1mg/Kg slow IV
• Repeat doses (2-3 times) based on clinical
response-
Seizures
Correct serum electrolyte abnormalities
More common
 Hyponatremia (lt125mg/Kg)
 5-10mL/Kg IV 3 saline solution
 Hypocalcaemia (lt7mg/dL)
 100-300mg/Kg IV of calcium gluconate

Seizures
 Immediately start broad-spectrum
antibiotics and acyclovir
 Neuroimaging once patient is
stabilized
 Admit to hospital for completion of
evaluation and monitoring
Conclusion
The mnemonic THE MISFITS
 Is a helpful tool
 Can be readily used to
formulate an approach to
 the most common neonatal
emergencies
Interventions & Outcomes in
ED
 Most needed only reassurance and advice,
no medication
 Of those Discharged from ED, only 1/3
needed investigations
More than half get discharged from ED
Alternatives/solutions
 24-hour clinics
 Walk-in clinic with paediatric specialist
 Teleconsultation
 24-hour telephone advice from pediatric nurse/Doctor
 Easier access to paediatricians
 Training GPs for community paediatrics
 Postnatal home visit by nurse or doctor
 Reading material provided during the birth hospital stay
 Information on the Internet
 information in newspapers/ magazines or on television/radio
Approaches to
Neonates in ED
Triage
Singapore Paediatric Triage Scale
Approaches to Neonates in ED
Past Medical History
History
Examination
Investigations
Past Medical History
 Antenatal and Obstetric history: fetal ultrasound
scans, Hx of miscarriage/NND
 Perinatal history: Term, PPROM/pyrexia, GBS status,
Liquor, Delivery
 Post natal course: Screening bloods, Blood groups,
G6PDD, medications, interventions, weight on
discharge
Clinical Presentation of septic neonates
Temperature Hyperthermia (more common in full
term infant)
Irregularity Hypothermia (more common than fever
in Pret
Skin Poor peripheral perfusion, cyanosis,
pallor, petechial, rashes
Gastrointestinal problems Feeding intolerance, vomiting,
diarrhea, or abdominal distention with
or without visible bowel loops
Cardiopulmonary Tachypnea, respiratory distress, apnea,
tachycardia and hypotension
Metabolic Hypoglycaemia, hyperglycaemia, or
metabolic acidosis
Focal infections Cellulitis, omphalitis, conjunctivitis,
otitis media or osteomyelitis
The patient
 “The norm”:
 Age — Young neonates (< 10 days old) who
present to the ED have a particularly high
incidence of serious illness, with 10% to 33%
requiring hospital admission.
 Feeds: type, quality and quantity
 Diaper output
 Weight
Feeding history
 Frequency and quality of breastfeeding
 How often and how much formula the bottle-fed infants
are receiving
 Frequency of stool and urination
 Check the weight of the neonate, a normal 10%
of weight loss in the first week is normal
 Regain birth weight at day 10-14
 Newborn gains 20-30 g per d
Presenting complaint: The
many faces of Sepsis
 Mostly subtle!
 Lethargy, irritability, and decreased oral intake
 Vomiting, diarrhea, temperature instability,
abdominal distention or ileus, apnea, tachypnea,
cyanosis, pallor, petechial and poor perfusion
 Look for the sources of infection
 Omphalitis
A two-day-old infant presents to the
emergency department with fever, poor
feeding, and irritability. She was born at
home at 38 weeks gestation. Which of
the following is the most likely pathogen
associated with early neonatal sepsis?
A) Enterococcus
B) Group B Streptococcus
C) Listeria monocytogenes
D) Staphylococcus aureus
MCQ time!
Physical exam
 Evaluation of neonatal size, weight
 Vital signs as heart rate, respiratory rate, oxygen
saturation level and temperature
 Pediatric Assessment triangle (PAT) provides an
accurate method for rapid assessment of any child
or infant with an emergency condition, through
visual and auditory evaluation of the infant’s
appearance, work of breathing and circulation to
skin
The Pediatric
Assessment
Triangle
 Rapid
 Global assessment tool
 Use visual and auditory clues
 Three key domains:
-appearance
-work of breathing
-circulation to the skin.
 The combination of
abnormalities determines the
category of pathophysiology
 Respiratory distress,
respiratory failure, CNS or
metabolic problem, shock, or
cardiopulmonary failure
The Pediatric Assessment
Triangle – Appearance- ‘TICLS’
 Tone – the newborn should have a normal flexed tone; the 6-
month-old baby who sits up and controls her head; the toddler
cruises around the room.
 Interactiveness – Does the 2-month-old have a social smile? Is the
toddler interested in what is going on in the room?
 Consolability – A child who cannot be consoled at some point by
his mother is experiencing a medical emergency until proven
otherwise.
 Look/gaze – Does the child track or fix his gaze on you, or is
there the “1000-yard stare”?
 Speech/cry – A vigorously crying baby can be a good sign, when
consolable – when the cry is high-pitched, blood-curling, or even
a soft whimper, something is wrong.
The Pediatric Assessment
Triangle- Work of Breathing
Symptom of breathlessness

Look for nasal flaring, retractions, abnormal positioning, like
tripodding, or head bobbing
 Listen – even without a stethoscope – for abnormal airway sounds
like grunting or stridor.
 Grunting is the child’s last-ditch effort to produce auto-PEEP.
 Stridor is a sign of critical upper airway narrowing.
The Pediatric Assessment
Triangle -Circulation to the skin
 Infants and children are vasospastic
 they can change their vascular tone quickly (depend on
volume status or environment)
Without even having to touch the child, you can see signs of:
-pallor,
-cyanosis,
-mottling.
 If any of these is present, this is an abnormal circulation to
the skin.
Assessment of ABC- Airway,
Breathing, Circulation
 Assessing the work of breathing and the respiratory rate
 A minute of observation or auscultation: assess the
respiratory rate and perceive the clinical signs of
difficulty in breathing.
 Presence of congenital heart disease: tachypnoea,
tachycardia, acrocyanosis, poor to absent peripheral
pulses and breathing difficulties, cardiac murmer
 Circulation and hydration: Capillary refill time is helpful
and should be less than 2-3 seconds.
 Hypotension is a late finding of shock; therefore
 Tachycardia must be taken seriously in the ill child
Physical exam
 VITAL SIGNS + blood sugar
 Cardio: Murmurs, perfusion (CRT, peripheral
colour), pulses, AFNT
 Resp: resp distress, cyanosis, percussion, BS & AE
 GI: distension, bowel loops, liver edge,
vomitus/feces, bowel sounds, umbilicus, Inguinal
region
 Neuro: primitive reflexes, equal movement, tone
 Skin
A limited neurological
examination in Neonates
 Evaluation of cry, tone, activity, cranial nerves, sensation
 Primitive reflexes
-Common primitive reflexes as rooting reflex, walking reflex, tonic
neck reflex, Moro reflex, palmar reflex and plantar reflex
 Seizures can present with apnea or changes in the tone alone
-Seizures may be due to glucose and electrolytes levels, central
nervous system infections, and metabolic disease or child abuse
 Muscular tone: healthy infants prefer to maintain his arms and
legs flexed.
-Hypotonic can be present in any critical ill infant,
-Hypertonia should be a concern for seizures or metabolic disease.
 Cry of the infant should be strong and vigorous.
-Neurological problems produce weak or shrill, high-pitched cry
Abdominal exam
 Newborn liver edge is palpable, but not spleen, palpable
spleen suspect hemolyzing states.
 Umbilical cord: signs of infection and bleeding.
 Constipation or bloody stool, fissure, NEC
 Genitals: hypospadias or other urethral anomalies,
inguinal hernias, varicoceles, hydroceles and
undescended testis.
 An incarcerated inguinal hernia can be seen as a hard
mass overlying the inguinal canal may be erythematous,
and it is usually tender to palpation
Skin conditions
 Most common benign rashes:
-Erythema toxicum whitish-yellow papules and pustules.
 Jaundice: Direct bilirubin levels must be measured,
Late onset jaundice
–Breastfeeding jaundice
-Ecoli UTI, Sepsis
-Metabolic disease
-Extrahepatic biliary atresia
Approach to the critically ill
neonate to ED
 Assessment of vital signs
 Placement of a cardiac monitor, pulse oximeter
 Bedside testing of blood glucose
 Temperature
 Delivery of oxygen (high flow mask)
 Difficult peripheral intravenous lines:
-Intra-osseous line, umbilical catheter must be placed.
 Blood glucose below 35-40 mg/dl is consider abnormal in
a neonate and should mandate an intervention
 Saline boluses are required10 ml per kg saline
Investigations
 FBC with IT ratio: >0.2 is 60 - 90% sensitive & 70%-
80% specific
 CRP: needs 8-10h
 Urea Electrolyte
 Ionized calcium
 Serum glucose
 Blood c/s
 ABG
 LP with CSF c/s
 In out urine
Action and disposition
 Commence IV antibiotics early!
 Control temperature
 Correct electrolytes
 All septic infants should be admitted to the hospital
WET FLAG
Weight (Age + 4 ) x 2 kg
Energy 4 J x Weight Joules
Tube
Internal Diameter = Age / 4 + 4 cm
Length (oral) = Age / 2 + 12 cm
Length (nasal) = Age /2 + 15 cm
Fluids
Medical = 20 ml x Weight
Trauma /bolus = 10 ml x Weight
Lorazepam
0.1mg x Weight nmg
Adrenaline
0.1ml x Weight of 1:10,000 Adrenaline mls
Glucose
2ml x Weight of 10% Dextrose mls
Special attention
Trauma- mostly
Trauma code
the leading cause of death and
disability in children
Unique paediatric differences
in trauma management
 Spine: SCIWORA
 Airway: relatively large tongue, large occiput, anterior
larynx, short trachea, needle cricothyroidotomy?
 Breathing: rib # less common, pulmonary injury without
#, Pneumothoraces, hypoxia is a more common cause of
arrest!, TV 6-8ml/kg
 Circulation: shock presents with tachycardia first (up to
35% loss of blood volume!), abdominal exsanguination,
insensible fluid losses, consider IO early
 Temperature management: more prone to hypothermia
Drowning
 In Singapore, death by drowning is the second
commonest cause of death due to unintentional injury in
children after road traffic injuries.
Unique things to consider in paediatrics
The following factors increase the risk of drowning:
 Epilepsy
 Cardiac dysrhythmias - congenital long QT syndrome,
catecholaminergic polymorphic ventricular
tachycardia and Brugada syndrome may be triggered
by swimming
 Hypoglycemia
 hyperventilation – can lead to syncope underwater
 Hypothermia resulting in body temperature less than
35°C – can cause poor muscle coordination and
weakness
 Trauma prior to drowning?
 Alcohol and illicit drugs – should be considered in
adolescents
Unique things to consider in
paediatrics
 Ventilation with lung protective measures reduces barotrauma and
should aim for normocapnia or mild hypocapnia. FiO2 should be
reduced to less than 0.5 as soon as possible to avoid pulmonary
oxygen toxicity.
 A study in Southern California found a poor outcome was likely for
a child with any of the following: • CPR in ED • apnoea and coma in
the ED • pH less than 7.0
 A RCT found targeted hypothermia (33°C) did not improve survival
or functional outcomes at 12 months when compared to
normothermia (36.8°C)
 Prophylactic antibiotics not recommended unless grossly
contaminated water
 all children who present with drowning, even if asymptomatic
should be observed for 4-8h for progression of pulmonary oedema
Non accidental injury
Powerful
statistics
In a study of 173 abused children with
head injuries, 54 were not recognized
as having been abused on initial
presentation. Fifteen of these
children were re-injured after the
missed diagnosis and four of these
children died
Mortality was significantly increased
in patients with recurrent NAT
compared with a single episode
(24.5% vs. 9.9%)
Risk factors for NAI
 Parents perception that there is little community support and when families feel a lack
of connection to the community
 decreased self-esteem, depression, history of suicide attempts, life stressors,
 parent in foster care or abandoned as a child
 unplanned or unwanted pregnancy
 engagement in criminal activity
 less prenatal care
 a history of relationship problems with other adults
 history of corporal punishment as a child
 shorter birth intervals
 increased number of separations from the child in the first year
 History of NAI in siblings
History taking in the ED
 Who comes with the child?
 Delays in seeking treatment … or an injury that looks
older than they say
 Inconsistent stories between historians
 Caregivers who have an inappropriate affect
 A pattern of injury that does not match what caregivers
say happened
 A child with a history of injuries
When to scan?
A skeletal survey should be obtained in the following
groups: any child less than 2 years of age with any
evidence of abuse, any child less than 5 years of age with a
suspicious fracture, or any older child who is unable to
communicate areas of pain or trauma (e.g., intellectually
disabled).
CT/ MRI brain should there be suspicion of intracranial
injury
CT without intravenous contrast remains the imaging
modality of choice for evaluating a child with acute
neurologic findings or RH on physical examination.
Differentials of NAI?
 Ask for family history of bleeding disorders
 Infections
 Accidental trauma
Shaken baby syndrome
 diagnosis is often missed since no history of head trauma
is provided
 non-specific presentation such as vomiting, poor
feeding, irritability or lethargy
 Classical injuries:
 Cranial: SDH
 Ocular manifestations of NAT: Retinal haemorrhage
 Long bone fractures
– Maguire et al
Clinical features of: apnea; retinal hemorrhage;
rib, skull and long-bone fractures; seizures and
head and/or neck bruising
If >3 of the clinical features were present, the
odds ratio was 100 and the positive predictive
value for abuse was above 85%
Subdural Haemangioma (SDH)
Differentials?
 Benign enlargement of the subdural space (BESS): rapid
increase in head circumference at 2-3 months of age
 head circumference > the 95th percentile at 3 years of age
 can still be diagnosed as SDH on CT or MRI brain
 Glutaric aciduria type I: rare autosomal recessive
neurometabolic disorder caused by a deficiency in glutarylCoA
dehydrogenase, which affects the degradation of lysine,
hydroxylysine, and tryptophan
 results in hypotonia, acute striatal necrosis, frontotemporal
atrophy and neurological deterioration
 present with macrocephaly and bilateral SDH
Neonatal Presentations to the Children’s Emergency Department
Sarah Blakey, Mark D Lyttle, Dan Magnus
medRxiv 2020.09.07.20190140; doi: https://doi.org/10.1101/2020.09.07.20
190140
Parental characteristics and perspectives pertaining to neonatal visits to
the emergency department: a multicentre survey.
Harrold J, Langevin M, Barrowman N, Sprague AE, Fell DB, Moreau KA,
Lacaze-Masmonteil T, Schuh S, Joubert G, Moore A, Solano T, Zemek RL;
Pediatric Emergency Research Canada Network. CMAJ Open. 2018 Sep
28;6(3):E423-E429. doi: 10.9778/cmajo.20180015. PMID: 30266780;
PMCID: PMC6182114.
Conclusion
 All emergency departments should be prepared to care for a critically
ill neonates, infant, having the appropriately sized equipment
 Nonemergency neonatal conditions should be managed in GP clinics
 The most common diagnosis in admitted neonates include
respiratory infections, sepsis, congenital heart disease, bowel
obstruction, hypoglycemia and seizures.
 Febrile neonates are at high risk for sepsis and therefore need
blood, urine and CSF testing, should receive empiric antibiotic
therapy in hospital
 Emergency physician must be trained to assess the neonate,
stabilize, narrow the differential diagnosis to the most likely and
begin life-sustaining treatment
 Many life-threatening conditions absent at birth, can present later in
Neonatal period with different clinical presentation, outcome can be
benign to catastrophic e.g. Congenital heart disease, metabolic,
sepsis
 Neonatal seizures may have subtle manifestations and require a
different approach than seizures in older infants and children
Q.1 Which of the following
statements is correct regarding this
infant’s fever?
A. The fever is unaffected by the mother’s bundling.
B. The mother should be instructed to alternate
acetaminophen and ibuprofen to better control the
fever.
C. Children with fever are at increased risk of brain
damage.
D. The fact that the fever responded to acetaminophen
indicates that a serious infection is less likely.
Q.2: In case of seizures due to
hypoglycemia in a newborn treat with
loading dose of the following:
A.20 mg/Kg of Phenobarbitone
B.2 ml/Kg of 10% Dextrose
C.5 ml/kg of 10% Dextrose
D.5 ml/Kg of 50% dextrose
Q.3: In case of shock in newborn
and If bleeding is not the likely
cause, then do the following
EXCEPT
A. Establish IV access
B. Give IV normal saline or ringer
lactate 10 ml/Kg over 10 minutes
C. Give IV normal saline or ringer
lactate 20 ml/Kg over 60 minutes
D. Give 10% Dextrose at
maintenance rate
Q.4:Which of the following
statements is TRUE concerning
Inborn Errors of Metabolism?
A. They are easy to diagnose
B. Symptoms always appear within
hours of birth
C. Initial symptoms are typically vague
and non-specific
D. They cannot be inherited
E. All of the statements are true
Q.5. A 2-day previously thought to be well and
was just discharged from the nursery in
morning, is presented as acutely pale,
cyanotic, with weak femoral and brachial
pulses. The congenital heart disease most
likely to present in this manner is
A. Tetralogy of Fallot
B. Hypoplastic Left Heart Syndrome
C. Tricuspid Atresia
D. Total Anomalous Pulmonary
Venous Return
E. Co-arctation of aorta
Q 6: A 10-day old male presents
with bilious emesis. What is the
most likely diagnosis?
A.Appendicitis
B.Pyloric stenosis
C.Malrotation with midgut
volvulus
D.Feeding intolerance
E.Necrotizing enterocolitis
Q.7: A 1-week-old male presents with some
mild erythema around his umbilicus extending
onto the abdominal wall. Which of the following
is the correct management for this patient?
A. Reassurance and continue with alcohol wipes of
umbilicus
B. Topical antibiotic ointment and recheck the patient
the next day
C. Discharge on cephalexin and recheck the next day
D. Perform a full septic workup and admit the patient
E. Apply silver nitrate pen to erythema to cauterize it
Q.8 : A 5-day old, well-appearing male is brought to
the ED by his mother today because she noted that he
has a cluster of vesicles on his scalp. Which of the
following should be the management approach?
A. Skin biopsy
B. IV acyclovir and a full septic
workup
C. Oral acyclovir
D. Discharge, with next day follow up
E. Refer to Dermatologist
Q.9: Which of the following heart rates is
most suggestive of supraventricular
tachycardia in a newborn?
180 BPM
230 BPM
150 BPM
210 BPM
Q.10: A 3-week-old male presenting to the emergency
department with vomiting and altered mental status
and acidosis. What additional laboratory test should be
included in your evaluation?
A. Ammonia level
B. Cortisol level
C. Serum acetone
D. Thyroid function test
E. CRP
Q.11: A 3-day-old female is
presented with vaginal bleeding
A. Is always indicative of child abuse
B. May be due to withdrawal of
maternal hormones
C. Is suspicious for gonorrhea
D. Is most commonly due to a vaginal
foreign body-such as baby wipes
Answer.1 Which of the following
statements is correct regarding this
infant’s fever?
A. The fever is unaffected by the mother’s bundling.
B. The mother should be instructed to alternate
acetaminophen and ibuprofen to better control the
fever.
C. Children with fever are at increased risk of brain
damage.
D. The fact that the fever responded to acetaminophen
indicates that a serious infection is less likely.
A.2: In case of seizures due to
hypoglycemia in a newborn treat with
loading dose of the following:
A.20 mg/Kg of Phenobarbitone
B.2 ml/Kg of 10% Dextrose
C.5 ml/kg of 10% Dextrose
D.5 ml/Kg of 50% dextrose
A.3: In case of shock in newborn and
If bleeding is not the likely cause,
then do the following immediately
EXCEPT
A. Establish IV access
B. Give IV normal saline or ringer
lactate 10 ml/Kg over 10 minutes
C. Give IV normal saline or ringer
lactate 20 ml/Kg over 60 minutes
D. Give 10% Dextrose at
maintenance rate
A.4:Which of the following
statements is TRUE concerning
Inborn Errors of Metabolism?
A. They are easy to diagnose
B. Symptoms always appear within hours
of birth
C. Initial symptoms are typically vague
and non-specific
D. They cannot be inherited
E. All of the statements are true
A.5. A 2-day previously thought to be well and
was just discharged from the nursery in morning,
is presented as acutely pale, cyanotic, with
weak femoral pulses. The congenital heart
disease most likely to present in this manner is
A. Tetralogy of Fallot
B. Hypoplastic Left Heart Syndrome
C. Tricuspid Atresia
D. Co-artctation of aorta
A 6: A 10-day old male presents
with bilious emesis. What is the
most likely diagnosis?
A. Appendicitis
B. Pyloric stenosis
C. Malrotation with midgut volvulus
D. Feeding intolerance
E. Necrotizing enterocolitis
A.7: A 1-week-old male presents with some mild
erythema around his umbilicus extending onto
the abdominal wall. Which of the following is the
correct management for this patient?
A. Reassurance and continue with alcohol wipes of
umbilicus
B. Topical antibiotic ointment and recheck the patient
the next day
C. Discharge on cephalexin and recheck the next day
D. Perform a full septic workup and admit the patient
E. Apply silver nitrate pen to erythema to cauterize it
A.8 : A 5-day old, well-appearing male is brought to
the ED by his mother today because she noted that he
has a cluster of vesicles on his scalp. Which of the
following should be the management approach?
A. Skin biopsy
B. IV acyclovir and a full septic
workup
C. Oral acyclovir
D. Discharge, with next day follow up
E. Refer to Dermatologist
A.9: Which of the following heart rates is
most suggestive of supraventricular
tachycardia in a newborn?
A. 180 BPM
B. 230 BPM
C. 150 BPM
D. 210 BPM
A.10: A 3-week-old male presenting to the emergency
department with vomiting and altered mental status
and acidosis. What additional laboratory test should be
included in your evaluation?
A. Ammonia level
B. Cortisol level
C. Serum acetone
D. Thyroid function test
E. CRP
A.11: A 3-day-old female is
presented with vaginal bleeding
A. Is always indicative of child abuse
B. Likely due to withdrawal of
maternal hormones
C. Is suspicious for gonorrhea
D. Is most commonly due to a vaginal
foreign body-such as baby wipes
E. Due to Vit K deficiency

More Related Content

What's hot

Neonatal fluid requirements and specials conditions
Neonatal fluid requirements and specials conditionsNeonatal fluid requirements and specials conditions
Neonatal fluid requirements and specials conditions
Rakesh Verma
 
Shock in Neonates
Shock in NeonatesShock in Neonates
Shock in Neonates
King_maged
 
Assessment And Managment Of Critically Ill Child 1
Assessment And Managment Of Critically Ill Child 1Assessment And Managment Of Critically Ill Child 1
Assessment And Managment Of Critically Ill Child 1
Dang Thanh Tuan
 
Fluids and electrolytes Newborns
Fluids and electrolytes NewbornsFluids and electrolytes Newborns
Fluids and electrolytes Newborns
Dr. Darayus P. Gazder
 
Fluid & electrolytes management in neonates
Fluid & electrolytes management in neonatesFluid & electrolytes management in neonates
Fluid & electrolytes management in neonates
Saurav Upadhyay
 
Neonatal emergencies guidelines
Neonatal emergencies guidelinesNeonatal emergencies guidelines
Neonatal emergencies guidelines
Sayed Ahmed
 
Pphn
PphnPphn
Management of a neonate with respiratory distress
Management of a neonate with respiratory distressManagement of a neonate with respiratory distress
Management of a neonate with respiratory distress
Soumya Ranjan Parida
 
Postnatal growth failure and its prevention
Postnatal growth failure and its preventionPostnatal growth failure and its prevention
Postnatal growth failure and its prevention
MCH-org-ua
 
Persistent pulmonary hypertension of newborn PPHN
Persistent pulmonary hypertension of newborn PPHNPersistent pulmonary hypertension of newborn PPHN
Persistent pulmonary hypertension of newborn PPHN
Chandan Gowda
 
Enteral nutrition in preterm neonates
Enteral nutrition in preterm neonatesEnteral nutrition in preterm neonates
Enteral nutrition in preterm neonates
Varsha Shah
 
Fluid and electrolyte management among neonates
Fluid and electrolyte management among neonatesFluid and electrolyte management among neonates
Fluid and electrolyte management among neonates
Geetanjli Kalyan
 
Surfactant admin
Surfactant adminSurfactant admin
Surfactant admin
lmcartagena
 
HIE-Pathophysiology & recent advances in management
HIE-Pathophysiology & recent advances in managementHIE-Pathophysiology & recent advances in management
HIE-Pathophysiology & recent advances in management
Viraj Satenahalli
 
Anemia of prematurity
Anemia of prematurityAnemia of prematurity
Anemia of prematurity
DrOdongRichardJustin
 
Neonatal sepsis
Neonatal sepsisNeonatal sepsis
Neonatal sepsis
Dr. Saad Saleh Al Ani
 
Fluid and electrolyte management in neonates. By Dr Rabab Hashem
Fluid and electrolyte management in neonates. By Dr Rabab HashemFluid and electrolyte management in neonates. By Dr Rabab Hashem
Fluid and electrolyte management in neonates. By Dr Rabab Hashem
mohamed osama hussein
 
Thyroid disorders in neonate radha
Thyroid disorders in neonate  radhaThyroid disorders in neonate  radha
Thyroid disorders in neonate radha
Dr Praman Kushwah
 
Neonatal shock
Neonatal shockNeonatal shock
Neonatal shock
. .
 
Respiratory distress in newborn
Respiratory distress in newbornRespiratory distress in newborn
Respiratory distress in newborn
All India Institute of Medical Sciences, Bhopal
 

What's hot (20)

Neonatal fluid requirements and specials conditions
Neonatal fluid requirements and specials conditionsNeonatal fluid requirements and specials conditions
Neonatal fluid requirements and specials conditions
 
Shock in Neonates
Shock in NeonatesShock in Neonates
Shock in Neonates
 
Assessment And Managment Of Critically Ill Child 1
Assessment And Managment Of Critically Ill Child 1Assessment And Managment Of Critically Ill Child 1
Assessment And Managment Of Critically Ill Child 1
 
Fluids and electrolytes Newborns
Fluids and electrolytes NewbornsFluids and electrolytes Newborns
Fluids and electrolytes Newborns
 
Fluid & electrolytes management in neonates
Fluid & electrolytes management in neonatesFluid & electrolytes management in neonates
Fluid & electrolytes management in neonates
 
Neonatal emergencies guidelines
Neonatal emergencies guidelinesNeonatal emergencies guidelines
Neonatal emergencies guidelines
 
Pphn
PphnPphn
Pphn
 
Management of a neonate with respiratory distress
Management of a neonate with respiratory distressManagement of a neonate with respiratory distress
Management of a neonate with respiratory distress
 
Postnatal growth failure and its prevention
Postnatal growth failure and its preventionPostnatal growth failure and its prevention
Postnatal growth failure and its prevention
 
Persistent pulmonary hypertension of newborn PPHN
Persistent pulmonary hypertension of newborn PPHNPersistent pulmonary hypertension of newborn PPHN
Persistent pulmonary hypertension of newborn PPHN
 
Enteral nutrition in preterm neonates
Enteral nutrition in preterm neonatesEnteral nutrition in preterm neonates
Enteral nutrition in preterm neonates
 
Fluid and electrolyte management among neonates
Fluid and electrolyte management among neonatesFluid and electrolyte management among neonates
Fluid and electrolyte management among neonates
 
Surfactant admin
Surfactant adminSurfactant admin
Surfactant admin
 
HIE-Pathophysiology & recent advances in management
HIE-Pathophysiology & recent advances in managementHIE-Pathophysiology & recent advances in management
HIE-Pathophysiology & recent advances in management
 
Anemia of prematurity
Anemia of prematurityAnemia of prematurity
Anemia of prematurity
 
Neonatal sepsis
Neonatal sepsisNeonatal sepsis
Neonatal sepsis
 
Fluid and electrolyte management in neonates. By Dr Rabab Hashem
Fluid and electrolyte management in neonates. By Dr Rabab HashemFluid and electrolyte management in neonates. By Dr Rabab Hashem
Fluid and electrolyte management in neonates. By Dr Rabab Hashem
 
Thyroid disorders in neonate radha
Thyroid disorders in neonate  radhaThyroid disorders in neonate  radha
Thyroid disorders in neonate radha
 
Neonatal shock
Neonatal shockNeonatal shock
Neonatal shock
 
Respiratory distress in newborn
Respiratory distress in newbornRespiratory distress in newborn
Respiratory distress in newborn
 

Similar to Neonatal presentations to Emergency department.pptx

Approach to chd
Approach to chdApproach to chd
Approach to chd
Bhadra Trivedi
 
Apparent Life Threatening Events
Apparent Life Threatening EventsApparent Life Threatening Events
Apparent Life Threatening Events
Rashid Abuelhassan
 
approach to sick neonate.pptx
approach to sick neonate.pptxapproach to sick neonate.pptx
approach to sick neonate.pptx
AbidJamali2
 
Brief resolved unexplained events (brue)
Brief resolved unexplained events (brue)Brief resolved unexplained events (brue)
Brief resolved unexplained events (brue)
Mohammad ALktifan
 
Pulse oximetry screening in newborns
Pulse oximetry screening in newbornsPulse oximetry screening in newborns
Pulse oximetry screening in newborns
gfalakha
 
Emergencies in gp
Emergencies in gpEmergencies in gp
Emergencies in gp
Chamath Fernando
 
Pediatrics History Taking and Physical Examination.pptx
Pediatrics History Taking and Physical Examination.pptxPediatrics History Taking and Physical Examination.pptx
Pediatrics History Taking and Physical Examination.pptx
AJAY MANDAL
 
The ed as gatekeeper in transitions of care james hoekstra md 1
The ed as gatekeeper in transitions of care james hoekstra md 1The ed as gatekeeper in transitions of care james hoekstra md 1
The ed as gatekeeper in transitions of care james hoekstra md 1
jgillmd
 
Prematurity (1).pptx
Prematurity (1).pptxPrematurity (1).pptx
Prematurity (1).pptx
احمد فياض
 
Hypoxic ischemic encephalopathy
Hypoxic ischemic encephalopathyHypoxic ischemic encephalopathy
Hypoxic ischemic encephalopathy
Matt Kimolo
 
After effects of bacterial meningitis and meningococcal disease: how and why
After effects of bacterial meningitis and meningococcal disease: how and why After effects of bacterial meningitis and meningococcal disease: how and why
After effects of bacterial meningitis and meningococcal disease: how and why
Meningitis Research Foundation
 
Management of HIE-1.pptx
Management of HIE-1.pptxManagement of HIE-1.pptx
Management of HIE-1.pptx
HafsaHussainp
 
Epilepsy CME Kisumu 10th February 2015
Epilepsy CME Kisumu 10th February 2015Epilepsy CME Kisumu 10th February 2015
Epilepsy CME Kisumu 10th February 2015
Aga Khan University Medical College of East Africa
 
Patent Ductus Arteriosus - management in preterm infants
Patent Ductus Arteriosus - management in preterm infantsPatent Ductus Arteriosus - management in preterm infants
Patent Ductus Arteriosus - management in preterm infants
Stefan Johansson
 
Prematurity.dr.leen
Prematurity.dr.leenPrematurity.dr.leen
Prematurity.dr.leen
LeenDoya
 
Clinical profile of paediatric patients with rheumatic heart disease at moi t...
Clinical profile of paediatric patients with rheumatic heart disease at moi t...Clinical profile of paediatric patients with rheumatic heart disease at moi t...
Clinical profile of paediatric patients with rheumatic heart disease at moi t...
Alexander Decker
 
Clinical profile of paediatric patients with rheumatic heart disease at moi t...
Clinical profile of paediatric patients with rheumatic heart disease at moi t...Clinical profile of paediatric patients with rheumatic heart disease at moi t...
Clinical profile of paediatric patients with rheumatic heart disease at moi t...
Alexander Decker
 
Evaluation of critically ill child
Evaluation of critically ill childEvaluation of critically ill child
Evaluation of critically ill child
Deiaa Tamer
 
Apparent life threatening event
Apparent life threatening eventApparent life threatening event
Apparent life threatening event
Dr. Abhinav Agarwal
 
FEBRILE SEIZURES -RAJEEV BAHALL
FEBRILE SEIZURES -RAJEEV BAHALLFEBRILE SEIZURES -RAJEEV BAHALL
FEBRILE SEIZURES -RAJEEV BAHALL
Rajeev Bahall
 

Similar to Neonatal presentations to Emergency department.pptx (20)

Approach to chd
Approach to chdApproach to chd
Approach to chd
 
Apparent Life Threatening Events
Apparent Life Threatening EventsApparent Life Threatening Events
Apparent Life Threatening Events
 
approach to sick neonate.pptx
approach to sick neonate.pptxapproach to sick neonate.pptx
approach to sick neonate.pptx
 
Brief resolved unexplained events (brue)
Brief resolved unexplained events (brue)Brief resolved unexplained events (brue)
Brief resolved unexplained events (brue)
 
Pulse oximetry screening in newborns
Pulse oximetry screening in newbornsPulse oximetry screening in newborns
Pulse oximetry screening in newborns
 
Emergencies in gp
Emergencies in gpEmergencies in gp
Emergencies in gp
 
Pediatrics History Taking and Physical Examination.pptx
Pediatrics History Taking and Physical Examination.pptxPediatrics History Taking and Physical Examination.pptx
Pediatrics History Taking and Physical Examination.pptx
 
The ed as gatekeeper in transitions of care james hoekstra md 1
The ed as gatekeeper in transitions of care james hoekstra md 1The ed as gatekeeper in transitions of care james hoekstra md 1
The ed as gatekeeper in transitions of care james hoekstra md 1
 
Prematurity (1).pptx
Prematurity (1).pptxPrematurity (1).pptx
Prematurity (1).pptx
 
Hypoxic ischemic encephalopathy
Hypoxic ischemic encephalopathyHypoxic ischemic encephalopathy
Hypoxic ischemic encephalopathy
 
After effects of bacterial meningitis and meningococcal disease: how and why
After effects of bacterial meningitis and meningococcal disease: how and why After effects of bacterial meningitis and meningococcal disease: how and why
After effects of bacterial meningitis and meningococcal disease: how and why
 
Management of HIE-1.pptx
Management of HIE-1.pptxManagement of HIE-1.pptx
Management of HIE-1.pptx
 
Epilepsy CME Kisumu 10th February 2015
Epilepsy CME Kisumu 10th February 2015Epilepsy CME Kisumu 10th February 2015
Epilepsy CME Kisumu 10th February 2015
 
Patent Ductus Arteriosus - management in preterm infants
Patent Ductus Arteriosus - management in preterm infantsPatent Ductus Arteriosus - management in preterm infants
Patent Ductus Arteriosus - management in preterm infants
 
Prematurity.dr.leen
Prematurity.dr.leenPrematurity.dr.leen
Prematurity.dr.leen
 
Clinical profile of paediatric patients with rheumatic heart disease at moi t...
Clinical profile of paediatric patients with rheumatic heart disease at moi t...Clinical profile of paediatric patients with rheumatic heart disease at moi t...
Clinical profile of paediatric patients with rheumatic heart disease at moi t...
 
Clinical profile of paediatric patients with rheumatic heart disease at moi t...
Clinical profile of paediatric patients with rheumatic heart disease at moi t...Clinical profile of paediatric patients with rheumatic heart disease at moi t...
Clinical profile of paediatric patients with rheumatic heart disease at moi t...
 
Evaluation of critically ill child
Evaluation of critically ill childEvaluation of critically ill child
Evaluation of critically ill child
 
Apparent life threatening event
Apparent life threatening eventApparent life threatening event
Apparent life threatening event
 
FEBRILE SEIZURES -RAJEEV BAHALL
FEBRILE SEIZURES -RAJEEV BAHALLFEBRILE SEIZURES -RAJEEV BAHALL
FEBRILE SEIZURES -RAJEEV BAHALL
 

More from Varsha Shah

pediatric emergencies
pediatric emergenciespediatric emergencies
pediatric emergencies
Varsha Shah
 
Pediatrics Examinations Made Easy for Medical Students by Dr Varsha 2023.docx
Pediatrics Examinations Made Easy for Medical Students by Dr Varsha 2023.docxPediatrics Examinations Made Easy for Medical Students by Dr Varsha 2023.docx
Pediatrics Examinations Made Easy for Medical Students by Dr Varsha 2023.docx
Varsha Shah
 
Examination in paediatric Medicine for medical students.pptx
Examination in paediatric Medicine for medical students.pptxExamination in paediatric Medicine for medical students.pptx
Examination in paediatric Medicine for medical students.pptx
Varsha Shah
 
Approach to thalassemia with abdominal distension in children
Approach to thalassemia  with abdominal distension in childrenApproach to thalassemia  with abdominal distension in children
Approach to thalassemia with abdominal distension in children
Varsha Shah
 
Jaundice in infant
Jaundice in infantJaundice in infant
Jaundice in infant
Varsha Shah
 
Approach to Cafe au lait spots in children
Approach to Cafe au lait spots in childrenApproach to Cafe au lait spots in children
Approach to Cafe au lait spots in children
Varsha Shah
 
Mcq in neonatology for medical students
Mcq in neonatology for medical studentsMcq in neonatology for medical students
Mcq in neonatology for medical students
Varsha Shah
 
Blood in stool in neonates
Blood in stool in neonatesBlood in stool in neonates
Blood in stool in neonates
Varsha Shah
 
Approach to cardiac murmurs and cardiac examination in children
Approach to cardiac murmurs and cardiac examination in childrenApproach to cardiac murmurs and cardiac examination in children
Approach to cardiac murmurs and cardiac examination in children
Varsha Shah
 
Developmental assessment for medical students
Developmental assessment for medical studentsDevelopmental assessment for medical students
Developmental assessment for medical students
Varsha Shah
 
[Mary sheridan] from_birth_to_five_years_children(bookos.org)[1]
[Mary sheridan] from_birth_to_five_years_children(bookos.org)[1][Mary sheridan] from_birth_to_five_years_children(bookos.org)[1]
[Mary sheridan] from_birth_to_five_years_children(bookos.org)[1]
Varsha Shah
 
7 breastfeeding the premature and the sick term baby
7 breastfeeding the premature and the sick term baby7 breastfeeding the premature and the sick term baby
7 breastfeeding the premature and the sick term baby
Varsha Shah
 
6 breastfeeding and drugs and acceptable medical reasons for artificial feedi...
6 breastfeeding and drugs and acceptable medical reasons for artificial feedi...6 breastfeeding and drugs and acceptable medical reasons for artificial feedi...
6 breastfeeding and drugs and acceptable medical reasons for artificial feedi...
Varsha Shah
 
5 breastfeeding for working mums
5 breastfeeding for working mums5 breastfeeding for working mums
5 breastfeeding for working mums
Varsha Shah
 
4 The rationale for skin to-skin contact at birth and rooming in
4 The rationale for skin to-skin contact at birth and rooming in4 The rationale for skin to-skin contact at birth and rooming in
4 The rationale for skin to-skin contact at birth and rooming in
Varsha Shah
 
4 rooming in and breast feeding
4 rooming in and breast feeding4 rooming in and breast feeding
4 rooming in and breast feeding
Varsha Shah
 
3 common breastfeeding challenges and its management
3 common breastfeeding challenges and its management3 common breastfeeding challenges and its management
3 common breastfeeding challenges and its management
Varsha Shah
 
2 physiology and benefits of bf, risk of artificial feeding230113
2 physiology and benefits of bf, risk of artificial feeding2301132 physiology and benefits of bf, risk of artificial feeding230113
2 physiology and benefits of bf, risk of artificial feeding230113
Varsha Shah
 
1 introduction to bfhi and 10 steps of breastfeeding
1 introduction to bfhi and 10 steps of breastfeeding1 introduction to bfhi and 10 steps of breastfeeding
1 introduction to bfhi and 10 steps of breastfeeding
Varsha Shah
 
Congenital diaphragmatic hernia by Dr. Varsha Atul Shah
Congenital diaphragmatic hernia by Dr. Varsha Atul ShahCongenital diaphragmatic hernia by Dr. Varsha Atul Shah
Congenital diaphragmatic hernia by Dr. Varsha Atul Shah
Varsha Shah
 

More from Varsha Shah (20)

pediatric emergencies
pediatric emergenciespediatric emergencies
pediatric emergencies
 
Pediatrics Examinations Made Easy for Medical Students by Dr Varsha 2023.docx
Pediatrics Examinations Made Easy for Medical Students by Dr Varsha 2023.docxPediatrics Examinations Made Easy for Medical Students by Dr Varsha 2023.docx
Pediatrics Examinations Made Easy for Medical Students by Dr Varsha 2023.docx
 
Examination in paediatric Medicine for medical students.pptx
Examination in paediatric Medicine for medical students.pptxExamination in paediatric Medicine for medical students.pptx
Examination in paediatric Medicine for medical students.pptx
 
Approach to thalassemia with abdominal distension in children
Approach to thalassemia  with abdominal distension in childrenApproach to thalassemia  with abdominal distension in children
Approach to thalassemia with abdominal distension in children
 
Jaundice in infant
Jaundice in infantJaundice in infant
Jaundice in infant
 
Approach to Cafe au lait spots in children
Approach to Cafe au lait spots in childrenApproach to Cafe au lait spots in children
Approach to Cafe au lait spots in children
 
Mcq in neonatology for medical students
Mcq in neonatology for medical studentsMcq in neonatology for medical students
Mcq in neonatology for medical students
 
Blood in stool in neonates
Blood in stool in neonatesBlood in stool in neonates
Blood in stool in neonates
 
Approach to cardiac murmurs and cardiac examination in children
Approach to cardiac murmurs and cardiac examination in childrenApproach to cardiac murmurs and cardiac examination in children
Approach to cardiac murmurs and cardiac examination in children
 
Developmental assessment for medical students
Developmental assessment for medical studentsDevelopmental assessment for medical students
Developmental assessment for medical students
 
[Mary sheridan] from_birth_to_five_years_children(bookos.org)[1]
[Mary sheridan] from_birth_to_five_years_children(bookos.org)[1][Mary sheridan] from_birth_to_five_years_children(bookos.org)[1]
[Mary sheridan] from_birth_to_five_years_children(bookos.org)[1]
 
7 breastfeeding the premature and the sick term baby
7 breastfeeding the premature and the sick term baby7 breastfeeding the premature and the sick term baby
7 breastfeeding the premature and the sick term baby
 
6 breastfeeding and drugs and acceptable medical reasons for artificial feedi...
6 breastfeeding and drugs and acceptable medical reasons for artificial feedi...6 breastfeeding and drugs and acceptable medical reasons for artificial feedi...
6 breastfeeding and drugs and acceptable medical reasons for artificial feedi...
 
5 breastfeeding for working mums
5 breastfeeding for working mums5 breastfeeding for working mums
5 breastfeeding for working mums
 
4 The rationale for skin to-skin contact at birth and rooming in
4 The rationale for skin to-skin contact at birth and rooming in4 The rationale for skin to-skin contact at birth and rooming in
4 The rationale for skin to-skin contact at birth and rooming in
 
4 rooming in and breast feeding
4 rooming in and breast feeding4 rooming in and breast feeding
4 rooming in and breast feeding
 
3 common breastfeeding challenges and its management
3 common breastfeeding challenges and its management3 common breastfeeding challenges and its management
3 common breastfeeding challenges and its management
 
2 physiology and benefits of bf, risk of artificial feeding230113
2 physiology and benefits of bf, risk of artificial feeding2301132 physiology and benefits of bf, risk of artificial feeding230113
2 physiology and benefits of bf, risk of artificial feeding230113
 
1 introduction to bfhi and 10 steps of breastfeeding
1 introduction to bfhi and 10 steps of breastfeeding1 introduction to bfhi and 10 steps of breastfeeding
1 introduction to bfhi and 10 steps of breastfeeding
 
Congenital diaphragmatic hernia by Dr. Varsha Atul Shah
Congenital diaphragmatic hernia by Dr. Varsha Atul ShahCongenital diaphragmatic hernia by Dr. Varsha Atul Shah
Congenital diaphragmatic hernia by Dr. Varsha Atul Shah
 

Recently uploaded

Can coffee help me lose weight? Yes, 25,422 users in the USA use it for that ...
Can coffee help me lose weight? Yes, 25,422 users in the USA use it for that ...Can coffee help me lose weight? Yes, 25,422 users in the USA use it for that ...
Can coffee help me lose weight? Yes, 25,422 users in the USA use it for that ...
nirahealhty
 
Michigan HealthTech Market Map 2024 with Policy Makers, Academic Innovation C...
Michigan HealthTech Market Map 2024 with Policy Makers, Academic Innovation C...Michigan HealthTech Market Map 2024 with Policy Makers, Academic Innovation C...
Michigan HealthTech Market Map 2024 with Policy Makers, Academic Innovation C...
Levi Shapiro
 
Anxiety, Trauma and Stressor Related Disorder.pptx
Anxiety, Trauma and Stressor Related Disorder.pptxAnxiety, Trauma and Stressor Related Disorder.pptx
Anxiety, Trauma and Stressor Related Disorder.pptx
Sagunlohala1
 
Bringing AI into a Mid-Sized Company: A structured Approach
Bringing AI into a Mid-Sized Company: A structured ApproachBringing AI into a Mid-Sized Company: A structured Approach
Bringing AI into a Mid-Sized Company: A structured Approach
Brian Frerichs
 
Let's Talk About It: Breast Cancer (What is Mindset and Does it Really Matter?)
Let's Talk About It: Breast Cancer (What is Mindset and Does it Really Matter?)Let's Talk About It: Breast Cancer (What is Mindset and Does it Really Matter?)
Let's Talk About It: Breast Cancer (What is Mindset and Does it Really Matter?)
bkling
 
TEST BANK FOR Health Assessment in Nursing 7th Edition by Weber Chapters 1 - ...
TEST BANK FOR Health Assessment in Nursing 7th Edition by Weber Chapters 1 - ...TEST BANK FOR Health Assessment in Nursing 7th Edition by Weber Chapters 1 - ...
TEST BANK FOR Health Assessment in Nursing 7th Edition by Weber Chapters 1 - ...
rightmanforbloodline
 
Gemma Wean- Nutritional solution for Artemia
Gemma Wean- Nutritional solution for ArtemiaGemma Wean- Nutritional solution for Artemia
Gemma Wean- Nutritional solution for Artemia
smuskaan0008
 
Stem Cell Solutions: Dr. David Greene's Path to Non-Surgical Cardiac Care
Stem Cell Solutions: Dr. David Greene's Path to Non-Surgical Cardiac CareStem Cell Solutions: Dr. David Greene's Path to Non-Surgical Cardiac Care
Stem Cell Solutions: Dr. David Greene's Path to Non-Surgical Cardiac Care
Dr. David Greene Arizona
 
Unlocking the Secrets to Safe Patient Handling.pdf
Unlocking the Secrets to Safe Patient Handling.pdfUnlocking the Secrets to Safe Patient Handling.pdf
Unlocking the Secrets to Safe Patient Handling.pdf
Lift Ability
 
Pediatric Emergency Care for Children | Apollo Hospital
Pediatric Emergency Care for Children | Apollo HospitalPediatric Emergency Care for Children | Apollo Hospital
Pediatric Emergency Care for Children | Apollo Hospital
Apollo 24/7 Adult & Paediatric Emergency Services
 
Professional Secrecy: Forensic Medicine Lecture
Professional Secrecy: Forensic Medicine LectureProfessional Secrecy: Forensic Medicine Lecture
Professional Secrecy: Forensic Medicine Lecture
DIVYANSHU740006
 
定制(wsu毕业证书)美国华盛顿州立大学毕业证学位证书实拍图原版一模一样
定制(wsu毕业证书)美国华盛顿州立大学毕业证学位证书实拍图原版一模一样定制(wsu毕业证书)美国华盛顿州立大学毕业证学位证书实拍图原版一模一样
定制(wsu毕业证书)美国华盛顿州立大学毕业证学位证书实拍图原版一模一样
khvdq584
 
TEST BANK For Accounting Information Systems, 3rd Edition by Vernon Richardso...
TEST BANK For Accounting Information Systems, 3rd Edition by Vernon Richardso...TEST BANK For Accounting Information Systems, 3rd Edition by Vernon Richardso...
TEST BANK For Accounting Information Systems, 3rd Edition by Vernon Richardso...
rightmanforbloodline
 
Deep Leg Vein Thrombosis (DVT): Meaning, Causes, Symptoms, Treatment, and Mor...
Deep Leg Vein Thrombosis (DVT): Meaning, Causes, Symptoms, Treatment, and Mor...Deep Leg Vein Thrombosis (DVT): Meaning, Causes, Symptoms, Treatment, and Mor...
Deep Leg Vein Thrombosis (DVT): Meaning, Causes, Symptoms, Treatment, and Mor...
The Lifesciences Magazine
 
Tips for Pet Care in winters How to take care of pets.
Tips for Pet Care in winters How to take care of pets.Tips for Pet Care in winters How to take care of pets.
Tips for Pet Care in winters How to take care of pets.
Dinesh Chauhan
 
KEY Points of Leicester travel clinic In London doc.docx
KEY Points of Leicester travel clinic In London doc.docxKEY Points of Leicester travel clinic In London doc.docx
KEY Points of Leicester travel clinic In London doc.docx
NX Healthcare
 
Rate Controlled Drug Delivery Systems.pdf
Rate Controlled Drug Delivery Systems.pdfRate Controlled Drug Delivery Systems.pdf
Rate Controlled Drug Delivery Systems.pdf
Rajarambapu College of Pharmacy Kasegaon Dist Sangli
 
CCSN_June_06 2024_jones. Cancer Rehabpptx
CCSN_June_06 2024_jones. Cancer RehabpptxCCSN_June_06 2024_jones. Cancer Rehabpptx
CCSN_June_06 2024_jones. Cancer Rehabpptx
Canadian Cancer Survivor Network
 
Top Rated Massage Center In Ajman Chandrima Spa
Top Rated Massage Center In Ajman Chandrima SpaTop Rated Massage Center In Ajman Chandrima Spa
Top Rated Massage Center In Ajman Chandrima Spa
Chandrima Spa Ajman
 
The Power of Superfoods and Exercise.pdf
The Power of Superfoods and Exercise.pdfThe Power of Superfoods and Exercise.pdf
The Power of Superfoods and Exercise.pdf
Dr Rachana Gujar
 

Recently uploaded (20)

Can coffee help me lose weight? Yes, 25,422 users in the USA use it for that ...
Can coffee help me lose weight? Yes, 25,422 users in the USA use it for that ...Can coffee help me lose weight? Yes, 25,422 users in the USA use it for that ...
Can coffee help me lose weight? Yes, 25,422 users in the USA use it for that ...
 
Michigan HealthTech Market Map 2024 with Policy Makers, Academic Innovation C...
Michigan HealthTech Market Map 2024 with Policy Makers, Academic Innovation C...Michigan HealthTech Market Map 2024 with Policy Makers, Academic Innovation C...
Michigan HealthTech Market Map 2024 with Policy Makers, Academic Innovation C...
 
Anxiety, Trauma and Stressor Related Disorder.pptx
Anxiety, Trauma and Stressor Related Disorder.pptxAnxiety, Trauma and Stressor Related Disorder.pptx
Anxiety, Trauma and Stressor Related Disorder.pptx
 
Bringing AI into a Mid-Sized Company: A structured Approach
Bringing AI into a Mid-Sized Company: A structured ApproachBringing AI into a Mid-Sized Company: A structured Approach
Bringing AI into a Mid-Sized Company: A structured Approach
 
Let's Talk About It: Breast Cancer (What is Mindset and Does it Really Matter?)
Let's Talk About It: Breast Cancer (What is Mindset and Does it Really Matter?)Let's Talk About It: Breast Cancer (What is Mindset and Does it Really Matter?)
Let's Talk About It: Breast Cancer (What is Mindset and Does it Really Matter?)
 
TEST BANK FOR Health Assessment in Nursing 7th Edition by Weber Chapters 1 - ...
TEST BANK FOR Health Assessment in Nursing 7th Edition by Weber Chapters 1 - ...TEST BANK FOR Health Assessment in Nursing 7th Edition by Weber Chapters 1 - ...
TEST BANK FOR Health Assessment in Nursing 7th Edition by Weber Chapters 1 - ...
 
Gemma Wean- Nutritional solution for Artemia
Gemma Wean- Nutritional solution for ArtemiaGemma Wean- Nutritional solution for Artemia
Gemma Wean- Nutritional solution for Artemia
 
Stem Cell Solutions: Dr. David Greene's Path to Non-Surgical Cardiac Care
Stem Cell Solutions: Dr. David Greene's Path to Non-Surgical Cardiac CareStem Cell Solutions: Dr. David Greene's Path to Non-Surgical Cardiac Care
Stem Cell Solutions: Dr. David Greene's Path to Non-Surgical Cardiac Care
 
Unlocking the Secrets to Safe Patient Handling.pdf
Unlocking the Secrets to Safe Patient Handling.pdfUnlocking the Secrets to Safe Patient Handling.pdf
Unlocking the Secrets to Safe Patient Handling.pdf
 
Pediatric Emergency Care for Children | Apollo Hospital
Pediatric Emergency Care for Children | Apollo HospitalPediatric Emergency Care for Children | Apollo Hospital
Pediatric Emergency Care for Children | Apollo Hospital
 
Professional Secrecy: Forensic Medicine Lecture
Professional Secrecy: Forensic Medicine LectureProfessional Secrecy: Forensic Medicine Lecture
Professional Secrecy: Forensic Medicine Lecture
 
定制(wsu毕业证书)美国华盛顿州立大学毕业证学位证书实拍图原版一模一样
定制(wsu毕业证书)美国华盛顿州立大学毕业证学位证书实拍图原版一模一样定制(wsu毕业证书)美国华盛顿州立大学毕业证学位证书实拍图原版一模一样
定制(wsu毕业证书)美国华盛顿州立大学毕业证学位证书实拍图原版一模一样
 
TEST BANK For Accounting Information Systems, 3rd Edition by Vernon Richardso...
TEST BANK For Accounting Information Systems, 3rd Edition by Vernon Richardso...TEST BANK For Accounting Information Systems, 3rd Edition by Vernon Richardso...
TEST BANK For Accounting Information Systems, 3rd Edition by Vernon Richardso...
 
Deep Leg Vein Thrombosis (DVT): Meaning, Causes, Symptoms, Treatment, and Mor...
Deep Leg Vein Thrombosis (DVT): Meaning, Causes, Symptoms, Treatment, and Mor...Deep Leg Vein Thrombosis (DVT): Meaning, Causes, Symptoms, Treatment, and Mor...
Deep Leg Vein Thrombosis (DVT): Meaning, Causes, Symptoms, Treatment, and Mor...
 
Tips for Pet Care in winters How to take care of pets.
Tips for Pet Care in winters How to take care of pets.Tips for Pet Care in winters How to take care of pets.
Tips for Pet Care in winters How to take care of pets.
 
KEY Points of Leicester travel clinic In London doc.docx
KEY Points of Leicester travel clinic In London doc.docxKEY Points of Leicester travel clinic In London doc.docx
KEY Points of Leicester travel clinic In London doc.docx
 
Rate Controlled Drug Delivery Systems.pdf
Rate Controlled Drug Delivery Systems.pdfRate Controlled Drug Delivery Systems.pdf
Rate Controlled Drug Delivery Systems.pdf
 
CCSN_June_06 2024_jones. Cancer Rehabpptx
CCSN_June_06 2024_jones. Cancer RehabpptxCCSN_June_06 2024_jones. Cancer Rehabpptx
CCSN_June_06 2024_jones. Cancer Rehabpptx
 
Top Rated Massage Center In Ajman Chandrima Spa
Top Rated Massage Center In Ajman Chandrima SpaTop Rated Massage Center In Ajman Chandrima Spa
Top Rated Massage Center In Ajman Chandrima Spa
 
The Power of Superfoods and Exercise.pdf
The Power of Superfoods and Exercise.pdfThe Power of Superfoods and Exercise.pdf
The Power of Superfoods and Exercise.pdf
 

Neonatal presentations to Emergency department.pptx

  • 1. Neonatal presentations to Emergency Department Dr Julia Jaffar, Medical Officer Supervised by Dr Varsha Atul Shah
  • 2. Neonatal presentations to Emergency Department Summary of presentation: 1. Background 2. Factors associated with Ed attendance among children included 3. Commonest neonatal presentations to ED 4. Singapore data 5. THE MISFITS-Acronym 6. Conclusions 7. CME-MCQ quiz
  • 3. Background:  ED attendances by neonates (≤28 days old) are rising disproportionately quicker than older infants  With shorter postpartum hospital stays, most of babies are discharges after 24 hours in normal vaginal delivery of 72 hours in LSCS  Many medical concerns, which traditionally have been managed in newborn nurseries, are presenting to the emergency department (ED)  Emergency Departments (ED) are under increasing pressure, with rising attendances and admission rates  Neonates present a challenge due to their non-specific presentations  Parents may preferentially bring their children to the ED, even for non-urgent problems  The reasons are parental anxiety, perceived advantages of Eds (resources and expertise), and convenience
  • 4. You are on call MO in SGH Emergency on busy day on Sunday Newborn well baby born to LSCS mother, was discharged on D3 of life on Saturday, with TCB of 192, was given routine NNJ advice to visit ED for repeat TCB next day. Presented to ED on busy Sunday afternoon for routine TCB check
  • 5. Questions are: 1. Mother with LSCS, was waiting in crowded ED next day, for routine check of TCB in crowded ED? 2. Is its appropriate use of resources? 3. Can they be better managed at 24 hours GP clinic? 4. Any other options?
  • 6. Case 2: Another extreme: 2 months old Exprem, who was discharged 3 days ago, was found lifeless with vomited blood in bed, sleeping with mother, was brought to ED by ambulance
  • 7. Questions is 1. Will ED will have enough time to attend to actual real emergency cases? 2. Is adult ED equipped and trained with managing such neonates? 2.Is it better to bring them to nearest Children’s Emergency dept?
  • 8.
  • 9. Daily attendance figures include all patients who are presented at the EMD regardless of urgency of medical conditions
  • 10. Singaporean context Unfortunately the most relevant SG paper on Neonatal presentations to ED is not specific to Neonates (classified as < 3 years) + the paper focusses on reattendances, not attendances “The patient’s age was found to be inversely proportional to the reattendance rate at our ED, consistent with preceding studies” Unplanned reattendances at the paediatric emergency department within 72 hours: a one-year experience in KKH - Goh G.L et al, SMJ 2016
  • 11. CGH ED study -Odds of emergency attendance in children and is inversely proportional to the age of the attendees Odds were 13 times more for <10 years than even > 70 years old
  • 12. Some factors that were reported to be associated with Ed attendance among children included Societal/Community: 1. The lack of a continuous relationship with PCPs 2. Caregivers overestimating the severity of their children’s illness 3. Caregivers having had prior experience of being taken to the ED as children 4. Being a single parent with no other adults living in the household 5. Availability of after-hour care offered by the ED 6. Mistrust of the PCP’s ability to manage paediatric conditions
  • 13. Factors associated with Ed attendance among children included Parental Parental anxiety Perception of convenience, perceived resources and expertise Believing the problem was more appropriate for or might require tests in the emergency department Feeling that the care provider would refer the baby to emergency department anyway
  • 14. Some factors that were reported to be associated with Ed attendance among children included Neonates  Neonates exhibit non-specific symptoms with serious conditions  Parental anxiety, lack of confidence, education, experience  Following discharge advice from nurses/doctors  Most are born at term, never separated from their mother and discharged within 48 hours of birth  Assessment of neonates is challenging  A vulnerable and high-risk cohort
  • 15. 1st problem: UK: ED attendances by neonates are rising disproportionately quicker than older infants In Europe and North America up to one-third of neonates attending EDs have no medical issue, and a high proportion have low acuity problems requiring no medical investigation or treatment
  • 16. Why act?  EDs are suboptimal environments for assessing well neonates  Potential over-investigation & treatment  Potentially modifiable group of patients whose problems can be managed away from the ED  Reducing ED attendances for neonates with benign conditions may benefit patients, families, and healthcare systems
  • 18.
  • 19.
  • 20. Commonest neonatal presentations Nonemergency  Neonatal Jaundice  Feeding problems, vomiting/regurgitation  Difficulty breathing,  Noisy breathing  Excessive crying/Colic  Irritability  ?Fever  Emergency • Respiratory • Cardiac • Shock • CNS
  • 21. Most common Diagnoses  More than half “No significant medical problem”  Bronchiolitis  ?Sepsis
  • 22. Common Neonatal Emergencies Acronym- The Misfits THE MISFITS  T- Trauma (accidental nonaccidental)  H- Heart Disease/Hypovolemia/Hypoxia  E- Endocrine (congenital adrenal hyperplasia,  thyrotoxicosis)  M- Metabolic (electrolyte imbalance)  I- Inborn Errors of Metabolism metabolic emergencies  S- Sepsis (meningitis, pneumonia, UTI)  F- Formula mishaps (under or over dilution)  I- Intestinal catastrophes (volvulus,  intussusception, NEC  T- Toxins/poisons  S- Seizures
  • 23. Trauma (accidental non-accidental)  May be a difficult process  Non-accidental? subtle historical findings and no  physical exam findings  Presenting symptoms may be nonspecific  Early diagnosis of an occult head injury may  prevent significant long-term morbidity  An ALTE is often an unrecognized presenting  symptom of abusive head injuries
  • 24. Trauma (accidental non- accidental)  Infants with ALTE w/o an immediate obvious cause should be evaluated for head trauma with neuroimaging  CT scan, HUS or MRI  Skull x-rays may not be helpful- significant head injury w/o skull fracture  Consider neuroimaging in any non-accidental injury for other skeletal injuries regardless of physical examination of the head 
  • 25. Trauma(accidental non- accidental)  37 of abused children lt 2 y/o had an occult traumatic injury  In addition, the ophthalmologic evaluation did not demonstrate retinal hemorrhages in most of the patients
  • 26. Trauma(accidental non-accidental) Management  Evaluation and stabilization of the ABCs  Bedside glucose evaluation  Appropriate temperature regulation If bruising or known intracranial bleed  CBC  Platelet count  PT/PTT  Neuroimaging after stabilization
  • 27. Trauma(accidental non-accidental)  Admit the patient  Report injury to appropriate state department for abuse  Skeletal survey  Ophthalmologic exam
  • 28. Heart Disease and Hypoxia Cyanotic Heart Disease Cyanosis requires immediate attention and evaluation Differential diagnosis  Respiratory causes  Infectious causes  CNS abnormalities  Toxins  Cyanotic heart disease
  • 29. Heart Disease and Hypoxia Cyanotic Heart Disease Terrible Ts  Transposition of the great arteries (TGA)  Tetralogy of Fallot (TOF)  Tricuspid atresia (TA)  Total anomalous pulmonary venous return (TAPVR)  Truncus arteriosus (TA)
  • 30. Heart Disease and Hypoxia Cyanotic Heart Disease • May not be detected at birth • Adequately oxygenated blood ? PDA ? systemic circulation • PDA functionally closes in the first 10-14 hrs of life • Several factors can delay its closure • Prematurity • Respiratory distress • Acidosis • Hypoxia 
  • 31. Heart Disease and Hypoxia Cyanotic Heart Disease  PDA is anatomically closed by 2 weeks of age, contributing to a delayed detection of cyanotic heart disease  100 FiO2  Non-cardiac disease  At least 10 increase in O2 saturation  Cyanotic heart disease  Minimal change in O2 saturation
  • 32. Heart Disease and Hypoxia Cyanotic Heart Disease • PDA is anatomically closed by 2 weeks of age, Hyperoxia test • Initial ABG on R/A • Repeat ABG after 10-20 minutes of 100 O2 • Cyanotic heart disease? PaO2 will not increase significantly • If PaO2 rises above 150 mm Hg, cardiac disease can generally be excluded • Failure of PaO2 to rise above 150 mm Hg suggests a cyanotic cardiac malformation 
  • 33. Heart Disease and Hypoxia Cyanotic Heart Disease  During stabilization, the physical exam should include 4 limb BP and careful cardiac exam  A murmur may be audible  Absence of a murmur does not exclude a cardiac defect  CXR EKG should be included in the evaluation  ECHO is diagnostic
  • 34. Heart Disease and Hypoxia Cyanotic Heart Disease Management  PGE1  Bolus of 0.05 mcg/Kg IV  Drip of 0.05-0.1 mcg/Kg/min  Secure airway  Profound apnea is a non-dose dependent  complication of PGE1
  • 35. Hypoplastic Left Heart Syndrome  25 of cardiac deaths during first week of life  Occurs in both cyanotic and acyanotic forms  In 15 of cases the FO is intact preventing  mixing at the atrial level  Infants with mixing at the atrial level are  acyanotic
  • 36. Hypoplastic Left Heart Syndrome  Pulmonary Edema  Pallor  Tachypnea  Poor perfusion  Poor to absent peripheral pulses  Loud single S2  Gallop rhythm w/o murmur  Hepatomegaly  Metabolic acidosis
  • 37. Hypoplastic Left Heart Syndrome  EKG  Small or absent (L) ventricular forces  CXR  Moderate cardiomegaly  Large PA shadow  ECHO  Small or slit-like (L) ventricle  Hypoplastic ascending aorta
  • 38. Hypoplastic Left Heart Syndrome- Treatment  PGE1- systemic blood flow is ductal dependent  Surgical correction  Surgical correction  1st stage  Norwood procedure  2nd stage  Fontan procedure  Neonatal cardiac transplantation  Compassionate care may be appropriate in some instances 
  • 39. Acyanotic Heart Disease- Congestive Heart Failure Typically presents with symptoms of CHF  Tachypnea  Tachycardia  Hepatomegaly  History of poor or slow feeding  Sweating or color change with feeding  Poor weight gain  More gradual clinical decompensation  May not present until after the first 2-3 weeks of age
  • 40. Acyanotic Heart Disease Congestive Heart Failure Causes of CHF in Neonates 1. Acyanotic heart disease (VSD, ASD, PDA, CoA) 2. Severe anemia 3. Trauma 4. Sepsis 5. SVT 6. Metabolic abnormalities 7. SLE 8. Thyrotoxicosis
  • 41. Acyanotic Heart Disease Congestive Heart Failure Initial management Stabilization of the ABCs CXR EKG Labs FBC BMP ABG ECHO- diagnostic of heart defect Furosemide-1 mg/Kg IV
  • 42. Acyanotic Heart Disease Congestive Heart Failure Pressors Dopamine 5-15 mcg/Kg/min IV Dobutamine 2.5-15 mcg/Kg/min IV Careful with fluid overloading Peds. Cardiology consult
  • 43. Acyanotic Heart Disease- Supraventricular Tachycardia Acyanotic Heart Disease  SVT is the most common neonatal dysrhythmia  (1/25,000 births) Signs/symptoms  Tachycardia  Poor feeding  Irritability  Heart Failure  Shock  Heart rate sustained at gt220 bpm with a QRS lt 0.08 seconds
  • 44. Acyanotic Heart Disease- Supraventricular Tachycardia Management  Stable patient  Vagal manoeuvres  Ice to face avoiding the nares  If unsuccessful  Adenosine  50 mcg/Kg rapid IVP (1-2 secs.), increase dose in 50mcg/Kg increments Q2 mins. until return of sinus rhythm, maximum dose 250 mcg/Kg
  • 45. Acyanotic Heart Disease- Supraventricular Tachycardia Management  Unstable patient w/o IV access  Synchronized cardioversion- 0.5-1 J/Kg  Initial cardioversion should be attempted pharmacologically if IV access is established and adenosine is readily available If unresponsive to adenosine cardioversion  Amiodorone- 5mg/Kg IV over 30-60 mins.
  • 46. Acyanotic Heart Disease- Supraventricular Tachycardia Procainamide- alternative to amiodorone- 15 mg/Kg IV over 30-60 mins.  The administration of procainamide and amiodorone together can lead to hypotension and widening of the QRS complex Lidocaine- 1mg/Kg IV Final option for a wide QRS and should only be used in consultation with a pediatric cardiologist 
  • 47. Acyanotic Heart Disease- Supraventricular Tachycardia  12-lead EKG prior to and after conversion from SVT to NSR  Useful diagnostic tool for the cardiologists to help determine further management  Consult pediatric cardiologist for further evaluation
  • 48. Heart Disease and Hypoxia Bronchiolitis- Viral  lower-airway disease caused by RSV 80 of the time  Other etiologies include adenovirus, influenza, or parainfluenza  RSV is responsible for 50-90 of bronchiolitis hospital admissions  More common in winter and spring seasons, may present at any time
  • 49. Heart Disease and Hypoxia Bronchiolitis- Viral Signs/Symptoms  Rhinorrhea  Cough  Congestion  Wheezing  Significant respiratory distress  Apnea may be the only initial symptom
  • 50. Heart Disease and Hypoxia Bronchiolitis- Viral Management  Infants with severe, prolonged apnea with bradycardia unresponsive to O2 therapy may need intubation  Nebulized racemic epinephrine or  Beta-agonist  The adjunct use of corticosteroids has not been shown to improve symptoms  A fever or sepsis evaluation may be part of the management
  • 51. Heart Disease and Hypoxia Bronchiolitis- Viral Management  Controversy over the incidence of severe bacterial infections in infants who have RSV  The presence of a viral infection doesnt exclude the possibility of a concomitant UTI  Consider hospitalization for all RSV() neonates, especially preemies or all neonates with other comorbidities
  • 52. Heart Disease and Hypoxia Apnea/ALTE  Apnea  cessation of respiration for 20 secs. or more, associated with color change (cyanosis or pallor) or bradycardia  ALTE poorly defined term used to describe any event that is frightening to the observer and is  characterized by some combination of apnea, color change, marked change in muscle tone, choking or gagging
  • 53. Heart Disease and Hypoxia Apnea/ALTE Management depends on history provided by observers and PE  Hospitalization for observation and monitoring  Common differential diagnosis  Sepsis  Pneumonia  RSV  Hypothermia  Anemia
  • 54. Heart Disease and Hypoxia Apnea/ALTE  Botulism  Dysrhythmias  Acid/base disturbances  Intracranial haemorrhage  Meningitis/encephalitis  Pertussis  Hypoglycaemia  Seizures  GER  Child abuse  Inborn errors of metabolism  Electrolyte abnormalities
  • 55. Endocrine Emergencies Congenital Adrenal Hyperplasia  Most patients diagnosed by newborn screening  Occasionally diagnosis is missed because of inadequate blood sample, laboratory error, or inability to contact the family
  • 56. Endocrine Emergencies Congenital Adrenal Hyperplasia  Autosomal recessive  Most common is 21-hydroxylase deficiency- 95 of affected patients  Inadequate cortisol levels  Excessive ACTH stimulation  Adrenal hyperplasia  Excessive production of adrenal androgens and testosterone  ? virilization 
  • 57. Endocrine Emergencies Congenital Adrenal Hyperplasia Two forms • Virilizing form • Relative aldosterone deficiency • Mild salt loss • Adrenal insufficiency tends not to occur unless under stressful situations • Salt-losing form • Absolute aldosterone deficiency • Adrenal insufficiency under basal conditions • Manifests in the neonatal period or soon after as an adrenal crisis 
  • 58. Endocrine Emergencies Congenital Adrenal Hyperplasia  11- hydroxylase deficiency  Less common- 5-8 of cases  Salt retention  Volume expansion  Hypertension 
  • 59. Endocrine Emergencies Congenital Adrenal Hyperplasia Management Labs  Blood glucose  Hypoglycaemia  Serum electrolytes  Hyponatremia  Hyperkalaemia  Hypotension unresponsive to fluids or inotropes heightens suspicion of CAH 
  • 60. Endocrine Emergencies Congenital Adrenal Hyperplasia Management  Hydrocortisone  25-50mg/m2 IV  Treat hypoglycemia  Hyperkalemia usually responds to fluid therapy  If patient is symptomatic or with EKG changes  Calcium chloride  NaHCO3  Insulin and glucose  Polystyrene sulfonate (Kayexalate)
  • 61. Endocrine Emergencies Congenital Adrenal Hyperplasia Management  Pediatric critical care management  Endocrinology consultation
  • 62. Endocrine Emergencies Thyrotoxicosis  Hypermetabolic state resulting from excessive thyroid hormone activity in the newborn  Usually results from transplacental passage of thyroid-stimulating immunoglobulin from a mother with Graves disease  Rare disorder  Occurs in 1/70 thyrotoxic pregnancies  Incidence of maternal thyrotoxicosis in pregnancy is 1-2/1000 pregnancies 
  • 63. Endocrine Emergencies Thyrotoxicosis Clinical presentation  Fetal tachycardia in the 3rd trimester may be the first manifestation  Signs usually apparent within hours from birth  If mother is on antithyroid medications presentation may be delayed 2-10 days  Thyrotoxic signs  Irritability  Tachycardia  Flushing  Tremor  Poor weight gain  Trombocytopenia  Arrhythmias
  • 64. Endocrine Emergencies Thyrotoxicosis  Initial diagnosis difficult w/o clear history of Graves disease from mother  Goiter usually present ? tracheal compression Labs  Increased T4, FT4 T3  Suppressed levels of TSH Treatment  Mild  Close observation 
  • 65. Endocrine Emergencies Thyrotoxicosis Moderate  Lugols solution (iodine)- 1 drop PO Q8H  Propylthiouracil- 5-10mg/Kg/day in 3 divided doses  Methimazole- 0.5-1mg/Kg/day in 3 divided doses Severe In addition to above meds  Prednisone- 2mg/Kg/day  Propranolol for tachycardia- 1-2mg/Kg/day in 2-4 divided doses  Digitalis may be used to prevent cardiovascular collapse
  • 66. Inborn Errors of Metabolism Urea cycle defects  Ornithine-transcarbamylase deficiency  Carbamyl phosphate synthetase deficiency  Transient hyperammonemia of the neonate (unclear cause)  Argininosuccinate synthetase deficiency (citrulinemia)  Argininosuccinate lyase deficiency  Arginase deficiency  N-acetylglutamate synthetase deficiency 
  • 67. Inborn Errors of Metabolism Amino acid metabolism defects  MSUD  Nonketotic hyperglycinaemia  Hereditary tyrosinemia  Pyroglutamic academia (5-oxoprolinuria)  Hyperornithinemia-hyperammonemia-homocitrulinemia syndrome  Lysinuric protein intolerance  Methylene tetrahydrofolate reductase deficiency  Sulphite oxidase deficiency
  • 68. Inborn Errors of Metabolism Organic Acidemias  Methylmalonic acidemia  Propionic acidemia  Isovaleric acidemia  Multiple carboxylase deficiency  Glutaric acidemia type II  HMG-CoA lyase deficiency  3-Memethylcrotonoyl-CoA carboxylase deficiency  3-Hydroxyisobutyric acidemia
  • 69. Inborn Errors of Metabolism Carbohydrate metabolism defects  Galactosemia  Fructose-1,6-biphosphatase deficiency  Glycogen storage diseases (types IA. IB, II, III and IV)  Hereditary fructose intolerance 
  • 70. Inborn Errors of Metabolism Fatty acid oxidation defects  Short chain acyl-CoA dehydrogenase deficiency (SCAD)  Medium chain acyl-CoA dehydrogenase deficiency (MCAD)  Most common (incidence of 1/6,000-10,000)  Long chain acyl-CoA dehydrogenase deficiency (LCAD)  Acyl-CoA deficiency 
  • 71. Inborn Errors of Metabolism- Metabolic Emergencies  Often have a delayed diagnosis  Symptoms may be unrecognized because they are  uncommon  Require a high level of suspicion for diagnosis  Diagnosis should be considered in any infant who  does not have any other obvious cause for symptoms
  • 72. Inborn Errors of Metabolism- Metabolic Emergencies  Nonspecific symptoms  Poor feeding  Vomiting  FTT  Tachycardia  Tachypnoea  Irritability 
  • 73. Inborn Errors of Metabolism- Metabolic Emergencies  More apparent symptoms  Seizures  Lethargy  Hypoglycemia  Apnea  Temperature instability  Acidosis 
  • 74. Inborn Errors of Metabolism- Metabolic Emergencies Labs  Bedside glucose  FBC  Basic metabolic panel (BMP)-UECr, Blood sugar, CaPo4  pH  Lactate and ammonia levels  LFTs  Urine for reducing substances and ketones  Blood and urine for organic and amino acids
  • 75. Inborn Errors of Metabolism- Metabolic Emergencies Management  Fluid resuscitation  IV dextrose to prevent further catabolism  Admission to hospital  Genetics consultation 
  • 76. Sepsis • It is standard of care to complete a full sepsis workup (FBC, blood culture, urinalysis, urine culture, CSF culture and analysis, CXR) in a neonate with a rectal temperature of 38 C 
  • 77. Sepsis  The use of peripheral WBC count is not helpful to differentiate febrile neonates with a more serious bacterial infection from those w/o serious bacterial infection  One study demonstrated that a low peripheral WBC count increased the odds of bacterial meningitis
  • 78. Sepsis • The urinalysis may be unremarkable in infants with a culture () UTI • Approximately 14 of febrile neonates will be diagnosed with a UTI • Pediatrics 2000 • McKay Memorial Hospital in Taiwan • CRP, ESR and U/A imperfect tools in discriminating for UTI 
  • 79. Sepsis Broad spectrum antibiotics  Ampicillin  50-100mg/Kg IV  Gentamicin  2mg/Kg IV or  Cefotaxime  50-100mg/Kg IV  Acyclovir  20mg/Kg IV 
  • 80. Sepsis Neonatal herpes • Symptoms may be subtle • No maternal history in 60-80 of women with unrecognized infection • Early recognition and treatment with acyclovir may decrease mortality from 90 ? 31 • Initiate treatment in any infant with • High fever • CSF lymphocytosis • Numerous RBCs in an atraumatic spinal tap • Seizures • Known maternal history of HSV infection 
  • 81. Sepsis  CSF analysis  Herpes PCR  Herpes culture  Elevated LFTs  Chest x-rays  Pneumonitis
  • 82. Formula Mishaps  Inappropriate mixing of water and powder formula  Overdilution of concentrated liquid or premixed formula  Life-threatening electrolyte disturbances or FTT  Hyponatremia  Seizures
  • 83. Intestinal Catastrophes  Consider pathologic process if vomiting in newborn period  Difficult to differentiate between a life-threatening cause from a mild viral gastroenteritis or even severe gatroesophageal reflux  Initial symptoms may be nonspecific  Bilious emesis is almost always an ominous sign  Initiate pediatric surgery consultation 
  • 84. Intestinal Catastrophes- Malrotation with Midgut Volvulus • Abnormal rotation of bowel in utero resulting in an unfixed portion of bowel that may later twist on itself ? bowel ischemia ? death • Incidence of 1/5,000 live births • Usually diagnosed in the first month of life 
  • 85. Intestinal Catastrophes- Malrotation with Midgut Volvulus Symptoms • Bilious emesis • Poor feeding • Lethargy • Shock in more advanced presentations • Management • Fluid resuscitation • NGT placement • Pediatric surgical consultation 
  • 86. Intestinal Catastrophes- Malrotation with Midgut Volvulus • KUBs-Normal • Signs of small bowel obstruction • Upper GI series is the gold standard for diagnosis • Transverse portion of the duodenum leading to a fixed ligament of Treitz 
  • 87. Intestinal Catastrophes- Toxic Megacolon  Life-threatening presentation of a patient with Hirschsprung's disease  Hirschsprung's disease occurs in 1/5,000 live births  May be unrecognized because constipation is common and usually benign  History of constipation with failure to pass meconium in the first 24 hours of life is highly suspicious of Hirschprungs 
  • 88. Intestinal Catastrophes- Toxic Megacolon Symptoms • Poor feeding • Vomiting • Irritability • Abdominal distention • Hematochezia • Shock as it progresses to enterocolitis 
  • 89. Intestinal Catastrophes- Toxic Megacolon Management • Stabilization of ABCs • Fluid resuscitation • Broad-spectrum antibiotics • KUB • Enlarged or dilated section of colon • Surgical consultation • Pediatric critical care management in the presence of enterocolitis 
  • 90. Intestinal Catastrophes- Necrotizing Enterocolitis  Classically a disease of premature infants  May occasionally occur in term neonates after discharge from WBN  Symptoms like those of Hirschsprung's enterocolitis 
  • 91. Intestinal Catastrophes- Necrotizing Enterocolitis Management  Stabilization of ABCs  Fluid resuscitation  NGT placement  Broad-spectrum antibiotics  Pediatric surgical consultation  Critical care management 
  • 92. Intestinal Catastrophes- Hypertrophic Pyloric Stenosis • Common, incidence of 1/250 live births • Male/female ratio 4/1 • More common in firstborn male • Classic metabolic abnormality of hypochloremic, hypokalemic metabolic alkalosis- now uncommon • History of nonbilious projectile emesis immediately after feeding 
  • 93. Intestinal Catastrophes- Hypertrophic Pyloric Stenosis  Increased incidence in infants with an early exposure to oral erythromycin PE  Palpable olive structure in the RUQ  Visible peristaltic waves  Diagnosis  USG  Thickened and lengthened pylorus  Upper GI  String sign
  • 94. Intestinal Catastrophes- Hypertrophic Pyloric Stenosis Management  Surgical is standard  IV atropine followed by oral atropine shows satisfactory results  Stabilization and IV access to replace fluids and electrolytes  Osaka, Japan  Archives of Disease in Childhood 2002  89 resolution of projectile vomiting with reduced pyloric muscle thickness
  • 95. Toxins  Toxic ingestions are uncommon  Occasionally the result of a maternal ingestion in a breastfeeding mother, homeopathic remedies, or overuse of accepted medications  Teething gels may be used for the relief of colic  Benzocaine  Methemoglobinemia with overuse
  • 96. Toxins  Star anise tea  Relief of infantile colic  Neurotoxicity  Unexplained irritability  Vomiting  Seizures  Baking soda  Used for intestinal gas  Serious toxicity  Hospitalization for monitoring and observation 
  • 97. Seizures  May be difficult to diagnose  Not acting right  More somnolent than usual  Immature cortical development  May not be tonic-clonic Commonly  Lip-smacking  Abnormal eye or tongue movements  Pedaling  Apnea
  • 98. Seizures Common causes of neonatal seizures 1st day of life  Anoxia/hypoxia  Trauma  Intracranial haemorrhage  Drugs  Infection  Hypoglycaemia/hyperglycaemia  Pyridoxine deficiency
  • 99. Seizures 2nd day of life  Sepsis  Trauma  Inborn errors of metabolism  Hypoglycaemia  Hypocalcaemia  Hyponatremia/hypernatremia  Hyperphosphatemia  Drug withdrawal  Congenital anomalies or developmental brain disorders  Benign familial neonatal seizures
  • 100. Seizures Day 4 to 6 months of age  Hypocalcaemia  Infection  Hyponatremia/hypernatremia  Drug withdrawal  Inborn errors of metabolism  Hyperphosphatemia  Congenital anomalies or developmental brain disorders  Hypertension  Benign idiopathic neonatal seizures 
  • 101. Seizures Management  Stabilization of ABCs  Labs  Bedside glucose level  Immediate correction of hypoglycemia (lt40mg/dL) with 2-4mL/Kg D10W may be necessary  Serum electrolytes  CBC  Blood CS  LFTs 
  • 102. Seizures Because 5-10 of neonatal seizures are of infectious etiology, full sepsis work-up should be performed when patient is stable
  • 103. Seizures Management • Phenobarbital • Loading dose 20mg/Kg slow IV push over 10-15 mins, additional 5mg/Kg doses up to 40mg/Kg • Maintenance of 3-4mg/Kg/day, 12-24 hours after loading dose 
  • 104. Seizures Phenytoin • Loading dose of 15-20mg/Kg IV over 30 minutes • Maintenance dose of 4-8mg/Kg IV slow push or PO • Highly unstable in IV solutions • Avoid using in central lines because of risk of precipitation • IM not an option- crystallizes in muscle Lorazepam - 0.05-0.1mg/Kg slow IV • Repeat doses (2-3 times) based on clinical response-
  • 105. Seizures Correct serum electrolyte abnormalities More common  Hyponatremia (lt125mg/Kg)  5-10mL/Kg IV 3 saline solution  Hypocalcaemia (lt7mg/dL)  100-300mg/Kg IV of calcium gluconate 
  • 106. Seizures  Immediately start broad-spectrum antibiotics and acyclovir  Neuroimaging once patient is stabilized  Admit to hospital for completion of evaluation and monitoring
  • 107. Conclusion The mnemonic THE MISFITS  Is a helpful tool  Can be readily used to formulate an approach to  the most common neonatal emergencies
  • 108. Interventions & Outcomes in ED  Most needed only reassurance and advice, no medication  Of those Discharged from ED, only 1/3 needed investigations More than half get discharged from ED
  • 109. Alternatives/solutions  24-hour clinics  Walk-in clinic with paediatric specialist  Teleconsultation  24-hour telephone advice from pediatric nurse/Doctor  Easier access to paediatricians  Training GPs for community paediatrics  Postnatal home visit by nurse or doctor  Reading material provided during the birth hospital stay  Information on the Internet  information in newspapers/ magazines or on television/radio
  • 110.
  • 112.
  • 114. Approaches to Neonates in ED Past Medical History History Examination Investigations
  • 115. Past Medical History  Antenatal and Obstetric history: fetal ultrasound scans, Hx of miscarriage/NND  Perinatal history: Term, PPROM/pyrexia, GBS status, Liquor, Delivery  Post natal course: Screening bloods, Blood groups, G6PDD, medications, interventions, weight on discharge
  • 116. Clinical Presentation of septic neonates Temperature Hyperthermia (more common in full term infant) Irregularity Hypothermia (more common than fever in Pret Skin Poor peripheral perfusion, cyanosis, pallor, petechial, rashes Gastrointestinal problems Feeding intolerance, vomiting, diarrhea, or abdominal distention with or without visible bowel loops Cardiopulmonary Tachypnea, respiratory distress, apnea, tachycardia and hypotension Metabolic Hypoglycaemia, hyperglycaemia, or metabolic acidosis Focal infections Cellulitis, omphalitis, conjunctivitis, otitis media or osteomyelitis
  • 117. The patient  “The norm”:  Age — Young neonates (< 10 days old) who present to the ED have a particularly high incidence of serious illness, with 10% to 33% requiring hospital admission.  Feeds: type, quality and quantity  Diaper output  Weight
  • 118. Feeding history  Frequency and quality of breastfeeding  How often and how much formula the bottle-fed infants are receiving  Frequency of stool and urination  Check the weight of the neonate, a normal 10% of weight loss in the first week is normal  Regain birth weight at day 10-14  Newborn gains 20-30 g per d
  • 119. Presenting complaint: The many faces of Sepsis  Mostly subtle!  Lethargy, irritability, and decreased oral intake  Vomiting, diarrhea, temperature instability, abdominal distention or ileus, apnea, tachypnea, cyanosis, pallor, petechial and poor perfusion  Look for the sources of infection  Omphalitis
  • 120. A two-day-old infant presents to the emergency department with fever, poor feeding, and irritability. She was born at home at 38 weeks gestation. Which of the following is the most likely pathogen associated with early neonatal sepsis? A) Enterococcus B) Group B Streptococcus C) Listeria monocytogenes D) Staphylococcus aureus MCQ time!
  • 121. Physical exam  Evaluation of neonatal size, weight  Vital signs as heart rate, respiratory rate, oxygen saturation level and temperature  Pediatric Assessment triangle (PAT) provides an accurate method for rapid assessment of any child or infant with an emergency condition, through visual and auditory evaluation of the infant’s appearance, work of breathing and circulation to skin
  • 122.
  • 123. The Pediatric Assessment Triangle  Rapid  Global assessment tool  Use visual and auditory clues  Three key domains: -appearance -work of breathing -circulation to the skin.  The combination of abnormalities determines the category of pathophysiology  Respiratory distress, respiratory failure, CNS or metabolic problem, shock, or cardiopulmonary failure
  • 124. The Pediatric Assessment Triangle – Appearance- ‘TICLS’  Tone – the newborn should have a normal flexed tone; the 6- month-old baby who sits up and controls her head; the toddler cruises around the room.  Interactiveness – Does the 2-month-old have a social smile? Is the toddler interested in what is going on in the room?  Consolability – A child who cannot be consoled at some point by his mother is experiencing a medical emergency until proven otherwise.  Look/gaze – Does the child track or fix his gaze on you, or is there the “1000-yard stare”?  Speech/cry – A vigorously crying baby can be a good sign, when consolable – when the cry is high-pitched, blood-curling, or even a soft whimper, something is wrong.
  • 125. The Pediatric Assessment Triangle- Work of Breathing Symptom of breathlessness  Look for nasal flaring, retractions, abnormal positioning, like tripodding, or head bobbing  Listen – even without a stethoscope – for abnormal airway sounds like grunting or stridor.  Grunting is the child’s last-ditch effort to produce auto-PEEP.  Stridor is a sign of critical upper airway narrowing.
  • 126. The Pediatric Assessment Triangle -Circulation to the skin  Infants and children are vasospastic  they can change their vascular tone quickly (depend on volume status or environment) Without even having to touch the child, you can see signs of: -pallor, -cyanosis, -mottling.  If any of these is present, this is an abnormal circulation to the skin.
  • 127. Assessment of ABC- Airway, Breathing, Circulation  Assessing the work of breathing and the respiratory rate  A minute of observation or auscultation: assess the respiratory rate and perceive the clinical signs of difficulty in breathing.  Presence of congenital heart disease: tachypnoea, tachycardia, acrocyanosis, poor to absent peripheral pulses and breathing difficulties, cardiac murmer  Circulation and hydration: Capillary refill time is helpful and should be less than 2-3 seconds.  Hypotension is a late finding of shock; therefore  Tachycardia must be taken seriously in the ill child
  • 128. Physical exam  VITAL SIGNS + blood sugar  Cardio: Murmurs, perfusion (CRT, peripheral colour), pulses, AFNT  Resp: resp distress, cyanosis, percussion, BS & AE  GI: distension, bowel loops, liver edge, vomitus/feces, bowel sounds, umbilicus, Inguinal region  Neuro: primitive reflexes, equal movement, tone  Skin
  • 129. A limited neurological examination in Neonates  Evaluation of cry, tone, activity, cranial nerves, sensation  Primitive reflexes -Common primitive reflexes as rooting reflex, walking reflex, tonic neck reflex, Moro reflex, palmar reflex and plantar reflex  Seizures can present with apnea or changes in the tone alone -Seizures may be due to glucose and electrolytes levels, central nervous system infections, and metabolic disease or child abuse  Muscular tone: healthy infants prefer to maintain his arms and legs flexed. -Hypotonic can be present in any critical ill infant, -Hypertonia should be a concern for seizures or metabolic disease.  Cry of the infant should be strong and vigorous. -Neurological problems produce weak or shrill, high-pitched cry
  • 130. Abdominal exam  Newborn liver edge is palpable, but not spleen, palpable spleen suspect hemolyzing states.  Umbilical cord: signs of infection and bleeding.  Constipation or bloody stool, fissure, NEC  Genitals: hypospadias or other urethral anomalies, inguinal hernias, varicoceles, hydroceles and undescended testis.  An incarcerated inguinal hernia can be seen as a hard mass overlying the inguinal canal may be erythematous, and it is usually tender to palpation
  • 131. Skin conditions  Most common benign rashes: -Erythema toxicum whitish-yellow papules and pustules.  Jaundice: Direct bilirubin levels must be measured, Late onset jaundice –Breastfeeding jaundice -Ecoli UTI, Sepsis -Metabolic disease -Extrahepatic biliary atresia
  • 132. Approach to the critically ill neonate to ED  Assessment of vital signs  Placement of a cardiac monitor, pulse oximeter  Bedside testing of blood glucose  Temperature  Delivery of oxygen (high flow mask)  Difficult peripheral intravenous lines: -Intra-osseous line, umbilical catheter must be placed.  Blood glucose below 35-40 mg/dl is consider abnormal in a neonate and should mandate an intervention  Saline boluses are required10 ml per kg saline
  • 133. Investigations  FBC with IT ratio: >0.2 is 60 - 90% sensitive & 70%- 80% specific  CRP: needs 8-10h  Urea Electrolyte  Ionized calcium  Serum glucose  Blood c/s  ABG  LP with CSF c/s  In out urine
  • 134. Action and disposition  Commence IV antibiotics early!  Control temperature  Correct electrolytes  All septic infants should be admitted to the hospital
  • 135. WET FLAG Weight (Age + 4 ) x 2 kg Energy 4 J x Weight Joules Tube Internal Diameter = Age / 4 + 4 cm Length (oral) = Age / 2 + 12 cm Length (nasal) = Age /2 + 15 cm Fluids Medical = 20 ml x Weight Trauma /bolus = 10 ml x Weight Lorazepam 0.1mg x Weight nmg Adrenaline 0.1ml x Weight of 1:10,000 Adrenaline mls Glucose 2ml x Weight of 10% Dextrose mls
  • 136. Special attention Trauma- mostly Trauma code the leading cause of death and disability in children
  • 137. Unique paediatric differences in trauma management  Spine: SCIWORA  Airway: relatively large tongue, large occiput, anterior larynx, short trachea, needle cricothyroidotomy?  Breathing: rib # less common, pulmonary injury without #, Pneumothoraces, hypoxia is a more common cause of arrest!, TV 6-8ml/kg  Circulation: shock presents with tachycardia first (up to 35% loss of blood volume!), abdominal exsanguination, insensible fluid losses, consider IO early  Temperature management: more prone to hypothermia
  • 138. Drowning  In Singapore, death by drowning is the second commonest cause of death due to unintentional injury in children after road traffic injuries.
  • 139. Unique things to consider in paediatrics The following factors increase the risk of drowning:  Epilepsy  Cardiac dysrhythmias - congenital long QT syndrome, catecholaminergic polymorphic ventricular tachycardia and Brugada syndrome may be triggered by swimming  Hypoglycemia  hyperventilation – can lead to syncope underwater  Hypothermia resulting in body temperature less than 35°C – can cause poor muscle coordination and weakness  Trauma prior to drowning?  Alcohol and illicit drugs – should be considered in adolescents
  • 140. Unique things to consider in paediatrics  Ventilation with lung protective measures reduces barotrauma and should aim for normocapnia or mild hypocapnia. FiO2 should be reduced to less than 0.5 as soon as possible to avoid pulmonary oxygen toxicity.  A study in Southern California found a poor outcome was likely for a child with any of the following: • CPR in ED • apnoea and coma in the ED • pH less than 7.0  A RCT found targeted hypothermia (33°C) did not improve survival or functional outcomes at 12 months when compared to normothermia (36.8°C)  Prophylactic antibiotics not recommended unless grossly contaminated water  all children who present with drowning, even if asymptomatic should be observed for 4-8h for progression of pulmonary oedema
  • 141.
  • 143. Powerful statistics In a study of 173 abused children with head injuries, 54 were not recognized as having been abused on initial presentation. Fifteen of these children were re-injured after the missed diagnosis and four of these children died Mortality was significantly increased in patients with recurrent NAT compared with a single episode (24.5% vs. 9.9%)
  • 144. Risk factors for NAI  Parents perception that there is little community support and when families feel a lack of connection to the community  decreased self-esteem, depression, history of suicide attempts, life stressors,  parent in foster care or abandoned as a child  unplanned or unwanted pregnancy  engagement in criminal activity  less prenatal care  a history of relationship problems with other adults  history of corporal punishment as a child  shorter birth intervals  increased number of separations from the child in the first year  History of NAI in siblings
  • 145. History taking in the ED  Who comes with the child?  Delays in seeking treatment … or an injury that looks older than they say  Inconsistent stories between historians  Caregivers who have an inappropriate affect  A pattern of injury that does not match what caregivers say happened  A child with a history of injuries
  • 146. When to scan? A skeletal survey should be obtained in the following groups: any child less than 2 years of age with any evidence of abuse, any child less than 5 years of age with a suspicious fracture, or any older child who is unable to communicate areas of pain or trauma (e.g., intellectually disabled). CT/ MRI brain should there be suspicion of intracranial injury CT without intravenous contrast remains the imaging modality of choice for evaluating a child with acute neurologic findings or RH on physical examination.
  • 147.
  • 148. Differentials of NAI?  Ask for family history of bleeding disorders  Infections  Accidental trauma
  • 149. Shaken baby syndrome  diagnosis is often missed since no history of head trauma is provided  non-specific presentation such as vomiting, poor feeding, irritability or lethargy  Classical injuries:  Cranial: SDH  Ocular manifestations of NAT: Retinal haemorrhage  Long bone fractures
  • 150. – Maguire et al Clinical features of: apnea; retinal hemorrhage; rib, skull and long-bone fractures; seizures and head and/or neck bruising If >3 of the clinical features were present, the odds ratio was 100 and the positive predictive value for abuse was above 85%
  • 151.
  • 152.
  • 153.
  • 154. Subdural Haemangioma (SDH) Differentials?  Benign enlargement of the subdural space (BESS): rapid increase in head circumference at 2-3 months of age  head circumference > the 95th percentile at 3 years of age  can still be diagnosed as SDH on CT or MRI brain  Glutaric aciduria type I: rare autosomal recessive neurometabolic disorder caused by a deficiency in glutarylCoA dehydrogenase, which affects the degradation of lysine, hydroxylysine, and tryptophan  results in hypotonia, acute striatal necrosis, frontotemporal atrophy and neurological deterioration  present with macrocephaly and bilateral SDH
  • 155. Neonatal Presentations to the Children’s Emergency Department Sarah Blakey, Mark D Lyttle, Dan Magnus medRxiv 2020.09.07.20190140; doi: https://doi.org/10.1101/2020.09.07.20 190140 Parental characteristics and perspectives pertaining to neonatal visits to the emergency department: a multicentre survey. Harrold J, Langevin M, Barrowman N, Sprague AE, Fell DB, Moreau KA, Lacaze-Masmonteil T, Schuh S, Joubert G, Moore A, Solano T, Zemek RL; Pediatric Emergency Research Canada Network. CMAJ Open. 2018 Sep 28;6(3):E423-E429. doi: 10.9778/cmajo.20180015. PMID: 30266780; PMCID: PMC6182114.
  • 156. Conclusion  All emergency departments should be prepared to care for a critically ill neonates, infant, having the appropriately sized equipment  Nonemergency neonatal conditions should be managed in GP clinics  The most common diagnosis in admitted neonates include respiratory infections, sepsis, congenital heart disease, bowel obstruction, hypoglycemia and seizures.  Febrile neonates are at high risk for sepsis and therefore need blood, urine and CSF testing, should receive empiric antibiotic therapy in hospital  Emergency physician must be trained to assess the neonate, stabilize, narrow the differential diagnosis to the most likely and begin life-sustaining treatment  Many life-threatening conditions absent at birth, can present later in Neonatal period with different clinical presentation, outcome can be benign to catastrophic e.g. Congenital heart disease, metabolic, sepsis  Neonatal seizures may have subtle manifestations and require a different approach than seizures in older infants and children
  • 157. Q.1 Which of the following statements is correct regarding this infant’s fever? A. The fever is unaffected by the mother’s bundling. B. The mother should be instructed to alternate acetaminophen and ibuprofen to better control the fever. C. Children with fever are at increased risk of brain damage. D. The fact that the fever responded to acetaminophen indicates that a serious infection is less likely.
  • 158. Q.2: In case of seizures due to hypoglycemia in a newborn treat with loading dose of the following: A.20 mg/Kg of Phenobarbitone B.2 ml/Kg of 10% Dextrose C.5 ml/kg of 10% Dextrose D.5 ml/Kg of 50% dextrose
  • 159. Q.3: In case of shock in newborn and If bleeding is not the likely cause, then do the following EXCEPT A. Establish IV access B. Give IV normal saline or ringer lactate 10 ml/Kg over 10 minutes C. Give IV normal saline or ringer lactate 20 ml/Kg over 60 minutes D. Give 10% Dextrose at maintenance rate
  • 160. Q.4:Which of the following statements is TRUE concerning Inborn Errors of Metabolism? A. They are easy to diagnose B. Symptoms always appear within hours of birth C. Initial symptoms are typically vague and non-specific D. They cannot be inherited E. All of the statements are true
  • 161. Q.5. A 2-day previously thought to be well and was just discharged from the nursery in morning, is presented as acutely pale, cyanotic, with weak femoral and brachial pulses. The congenital heart disease most likely to present in this manner is A. Tetralogy of Fallot B. Hypoplastic Left Heart Syndrome C. Tricuspid Atresia D. Total Anomalous Pulmonary Venous Return E. Co-arctation of aorta
  • 162. Q 6: A 10-day old male presents with bilious emesis. What is the most likely diagnosis? A.Appendicitis B.Pyloric stenosis C.Malrotation with midgut volvulus D.Feeding intolerance E.Necrotizing enterocolitis
  • 163. Q.7: A 1-week-old male presents with some mild erythema around his umbilicus extending onto the abdominal wall. Which of the following is the correct management for this patient? A. Reassurance and continue with alcohol wipes of umbilicus B. Topical antibiotic ointment and recheck the patient the next day C. Discharge on cephalexin and recheck the next day D. Perform a full septic workup and admit the patient E. Apply silver nitrate pen to erythema to cauterize it
  • 164. Q.8 : A 5-day old, well-appearing male is brought to the ED by his mother today because she noted that he has a cluster of vesicles on his scalp. Which of the following should be the management approach? A. Skin biopsy B. IV acyclovir and a full septic workup C. Oral acyclovir D. Discharge, with next day follow up E. Refer to Dermatologist
  • 165. Q.9: Which of the following heart rates is most suggestive of supraventricular tachycardia in a newborn? 180 BPM 230 BPM 150 BPM 210 BPM
  • 166. Q.10: A 3-week-old male presenting to the emergency department with vomiting and altered mental status and acidosis. What additional laboratory test should be included in your evaluation? A. Ammonia level B. Cortisol level C. Serum acetone D. Thyroid function test E. CRP
  • 167. Q.11: A 3-day-old female is presented with vaginal bleeding A. Is always indicative of child abuse B. May be due to withdrawal of maternal hormones C. Is suspicious for gonorrhea D. Is most commonly due to a vaginal foreign body-such as baby wipes
  • 168. Answer.1 Which of the following statements is correct regarding this infant’s fever? A. The fever is unaffected by the mother’s bundling. B. The mother should be instructed to alternate acetaminophen and ibuprofen to better control the fever. C. Children with fever are at increased risk of brain damage. D. The fact that the fever responded to acetaminophen indicates that a serious infection is less likely.
  • 169. A.2: In case of seizures due to hypoglycemia in a newborn treat with loading dose of the following: A.20 mg/Kg of Phenobarbitone B.2 ml/Kg of 10% Dextrose C.5 ml/kg of 10% Dextrose D.5 ml/Kg of 50% dextrose
  • 170. A.3: In case of shock in newborn and If bleeding is not the likely cause, then do the following immediately EXCEPT A. Establish IV access B. Give IV normal saline or ringer lactate 10 ml/Kg over 10 minutes C. Give IV normal saline or ringer lactate 20 ml/Kg over 60 minutes D. Give 10% Dextrose at maintenance rate
  • 171. A.4:Which of the following statements is TRUE concerning Inborn Errors of Metabolism? A. They are easy to diagnose B. Symptoms always appear within hours of birth C. Initial symptoms are typically vague and non-specific D. They cannot be inherited E. All of the statements are true
  • 172. A.5. A 2-day previously thought to be well and was just discharged from the nursery in morning, is presented as acutely pale, cyanotic, with weak femoral pulses. The congenital heart disease most likely to present in this manner is A. Tetralogy of Fallot B. Hypoplastic Left Heart Syndrome C. Tricuspid Atresia D. Co-artctation of aorta
  • 173. A 6: A 10-day old male presents with bilious emesis. What is the most likely diagnosis? A. Appendicitis B. Pyloric stenosis C. Malrotation with midgut volvulus D. Feeding intolerance E. Necrotizing enterocolitis
  • 174. A.7: A 1-week-old male presents with some mild erythema around his umbilicus extending onto the abdominal wall. Which of the following is the correct management for this patient? A. Reassurance and continue with alcohol wipes of umbilicus B. Topical antibiotic ointment and recheck the patient the next day C. Discharge on cephalexin and recheck the next day D. Perform a full septic workup and admit the patient E. Apply silver nitrate pen to erythema to cauterize it
  • 175. A.8 : A 5-day old, well-appearing male is brought to the ED by his mother today because she noted that he has a cluster of vesicles on his scalp. Which of the following should be the management approach? A. Skin biopsy B. IV acyclovir and a full septic workup C. Oral acyclovir D. Discharge, with next day follow up E. Refer to Dermatologist
  • 176. A.9: Which of the following heart rates is most suggestive of supraventricular tachycardia in a newborn? A. 180 BPM B. 230 BPM C. 150 BPM D. 210 BPM
  • 177. A.10: A 3-week-old male presenting to the emergency department with vomiting and altered mental status and acidosis. What additional laboratory test should be included in your evaluation? A. Ammonia level B. Cortisol level C. Serum acetone D. Thyroid function test E. CRP
  • 178. A.11: A 3-day-old female is presented with vaginal bleeding A. Is always indicative of child abuse B. Likely due to withdrawal of maternal hormones C. Is suspicious for gonorrhea D. Is most commonly due to a vaginal foreign body-such as baby wipes E. Due to Vit K deficiency

Editor's Notes

  1. anyone can comment some issues with the scenario itself?
  2. Tyebally A, Ang S Y 2010 - SMJ
  3. as such the history taking should be targeted around this
  4. Queensland paediatric guide in management of drowning in children
  5. bolded are risk factors that we can already identify in the neonatal period Paul AR, Adamo MA. Non-accidental trauma in pediatric patients: a review of epidemiology, pathophysiology, diagnosis and treatment. Transl Pediatr. 2014;3(3):195-207. doi:10.3978/j.issn.2224-4336.2014.06.01
  6. Accidental bruises are commonly over bony prominences Also not if baby is not cruising A high percentage of childhood burns are due to abuse (2% to 35% overall; up to 45% for genital and perineal burns). The two kinds of burns most often seen in abused children are scald burns (from contact with hot liquids) and thermalburns (contact with hot objects). Spiral fractures, indicative of a twist injury, of the humeral shaft are significantly more common in abused children
  7. Children’s sutures may not have closed, hematoma able to expand into space Morechildren in the accidental trauma group presented with skull fracture (57%) whereas patients in the NAT group were more likely to present with SDH (52%), periorbital hematoma, eyelid laceration, subconjunctival hemorrhage, subluxed or dislocated lens, cataracts, glaucoma, anterior chamber angle regression, iridiodialysis, retinal dialysis or detachment, intraocular hemorrhage, optic atrophy or papilledema The sensitivity of RH was estimated to be 75% and the specificity at 93.2% for the diagnosis of child abuse; the predictive positive and negative values were 89.4% and 82.9%, respectively
  8. Arch Dis Child Educ Pract Ed . 2010 Dec;95(6):170-7. doi: 10.1136/adc.2009.170431. Epub 2010 Oct 6.
  9. healed framers over R ribs newer fractures over L ribs
  10. Radiographic findings commonly seen in non-accidental trauma (NAT). (A) A left frontoparietal epidural hematoma in a 9-monthold male after being dropped; (B) an acute left frontoparietal subdural hematoma in a 14-year-old male following an assault; (C) a small amount of traumatic subarachnoid hemorrhage in the right sylvian fissure in a 12-year-old male following assault; (D) a small left frontal contusion in a 31-day-old female who was dropped; (E) restricted diffusion in the left thalamus, splenium of the corpus callosum and bilateral occipital poles in a 14-year-old female; (F) right frontal epidural hemorrhage as well as bifrontal and right posteromedial parietal/ occipital lobe hypodensity representing infarction in a 3-year-old male following abuse